Головна · Гастрит · Тести з клінічної фармакології для студентів. Тести з клінічної фармакології для студентів курсу педіатричного факультету. Тема: Способи та види дії фармакологічних засобів

Тести з клінічної фармакології для студентів. Тести з клінічної фармакології для студентів курсу педіатричного факультету. Тема: Способи та види дії фармакологічних засобів

КЛІНІЧНА фармакологія

001. Як змінюється обсяг розподілу жиророзчинних ЛЗ

у опасистих хворих?

а) зменшується

в) не змінюється

г) не змінюється чи збільшується

д) збільшується

002. Як змінюється біотрансформація ЛЗ на фоні куріння та прийому алкоголю?

а) зменшується

б) зменшується або не змінюється

в) не змінюється

г) не змінюється чи посилюється

д) посилюється

003. Від якого основного чинника залежить біоеквівалентність ліків?

а) фармакодинамічні характеристики ЛЗ

б) фізико-хімічні характеристики

в) лікарська форма

г) технологія виготовлення

д) стан організму пацієнта

004. Які побічні дії ЛЗ не залежать від дози?

а) пов'язані з фармакологічними властивостями ЛЗ

б) токсичні ускладнення,

обумовлені абсолютним чи відносним передозуванням

в) вторинні ефекти, зумовлені порушенням

імунобіологічних властивостей організму

г) імунологічні реакції негайного та уповільненого типів

д) синдром відміни

005. Яке ускладнення у новонароджених

Чи може викликати введення вагітним перед пологами сульфату магнію?

а) розвиток нейро-м'язових блокад та летаргії

б) пригнічення дихання

в) тромбоцитопенію

г) гепатотоксична дія

д) гіпотрофію

006. Які ускладнення у новонароджених

може викликати призначення вагітним -адреноблокаторів?

а) втрата слуху

б) ураження шкірних покривів

в) гіпотрофію плаценти та плода

г) передчасне закриття боталової протоки

д) геморагічний синдром

007. Застосування якихось антимікробних препаратів

найбезпечніше під час вагітності?

а) аміноглікозиди

б) котримоксазол

в) нітрофурани

г) пеніциліни

д) фторхінолони

008. Вкажіть антибіотик, який не має антисиньогнійної активності:

а) карбеніцилін

б) ампіокс

в) тикарцилін

д) азлоцилін

г) цефтазидим

009. Хворий на пневмонію, що отримує антибактеріальне лікування,

став пред'являти скарги на запаморочення,

нестійкість та похитування при ходьбі.

Застосування якого антибіотика могло спричинити ці симптоми?

а) ампіцилін

б) цефоперазон

в) гентаміцин

г) еритроміцин

д) лінкоміцин

010. Вкажіть антибіотик, який є препаратом вибору

при лікуванні інфекцій, спричинених staph. aureus:

а) пеніцилін

б) гентаміцин

в) азитроміцин

г) левоміцетин

д) амоксицилін-клавуланат

011. Вкажіть антибіотик, який є препаратом вибору

при лікуванні інфекцій,

викликаних метицилінрезистентним стафілококом:

а) лінкоміцин

б) еритроміцин

в) ванкоміцин

г) пеніцилін

д) оксацилін

012. Вкажіть антибіотик,

а) ампіцилін

б) гентаміцин

в) цефоперазон

г) метранідозол

д) тетрациклін

013. Активний проти атипових збудників

(мікоплазма, хламідії, легіонелла):

а) гентаміцин

б) еритроміцин

в) ампіокс

г) левоміцетин

д) кліндаміцин

014. Котримоксазол є препаратом вибору при лікуванні:

а) пневмоцистної пневмонії у хворих на імунодефіцит

б) дифтерії

в) холангіту

г) пневмококової пневмонії

д) амебної дизентерії

015. Який антибіотик протипоказаний хворим,

одержуючим міорелаксанти або при міастенії?

а) ампіцилін

б) гентаміцин

в) еритроміцин

г) лінкоміцин

д) ципрофлоксацин

016. Хворому на бронхіальну астму,

що отримує постійно преднізолон всередину, теопек, інгаляції беротека,

у зв'язку з бронхолегеневою інфекцією, що приєдналася.

були призначені еритроміцин та бромгексин.

На третій день лікування у хворого з'явився головний біль, занепокоєння,

дратівливість, серцебиття, відчуття перебоїв у серці,

зниження артеріального тиску, підвищення температури, нудота, блювання.

З токсичною дією якого препарату пов'язані ці симптоми?

а) Преднізолон

б) теопек

в) еритроміцин

г) беротек

д) бромгексин

017. Концентрацію теофіліну в крові знижують усі зазначені препарати,

а) фенобарбіталу

б) рифампіцину

в) карбамазипіну

г) ніфедипіну

д) фенітоїну

018. Середня терапевтична концентрація теофіліну в плазмі становить:

а) 10-20 мкг/мл

б) 25-30 мкг/мл

в) 30-35 мкг/мл

г) 35-40 мкг/мл

д) 5-10 мкг/мл

019. При поєднаній дії теофіліну та циметидину дія еуфіліну:

а) посилюється

б) посилюється чи змінюється

в) зменшується

г) зменшується або не змінюється

д) не змінюється

020. Тривалість дії ксантинів:

а) 1-2 години

б) 2-3 години

в) 3-4 години

г) 6-8 годин

д) 10-12 годин

021. Який механізм дії агоністів 2-адренорецепторів?

а) інгібування фосфодіестерази

б) гальмування дегрануляції опасистих клітин

в) блокада гістамінових рецепторів

г) пригнічення дії лейкотрієнів на дихальні шляхи

д) активація аденілатциклази, збільшення освіти ЦАМФ

022. Вкажіть початок, максимум дії та тривалість дії

фенотеролу (беротека):

а) негайно, 10 хвилин, 6 годин

б) 15 хвилин, 30 хвилин, 6 годин

в) 2-3 хвилин, 20 хвилин, 2,5 години

г) 5-10 хвилин, 30 хвилин, 6 годин

д) 30-40 секунд, 20 хвилин, 3-5 годин

023. Вкажіть найчастіші побічні ефекти

інгаляторних форм глюкокортикоїдів:

а) розвиток остеопорозу

б) гіперкортицизм

в) кандидоз порожнини рота та глотки

г) артеріальна гіпертонія

024. На відміну від беклометазону пропіонату, будезонід має:

а) більш значним спорідненістю до рецепторів у легенях,

піддається активній біотрансформації у печінці

при першому проходженні

б) більшою мірою пригнічує утворення гідрокортизону

в) найчастіше веде до розвитку гіперглікемії

г) частіше викликає загострення бронхо-легеневої інфекції

д) відмінність між препаратами відсутня

025. Вкажіть 2-адреноміметик тривалої дії:

а) сальбутамол

б) тербуталін

в) фенотерол

г) орципреналіну сульфат

д) формотерол

026. Вкажіть препарат, що має найбільш потужну пригнічуючу дію

на шлункову секрецію:

а) омепразол

б) циметидин

в) фамотидин

г) сукральфат

для лікування рецидиву виразкової хвороби 12-палої кишки:

028. Хворий, який страждає на ревматоїдний поліартрит,

тривалий час отримує НПЗП.

Який препарат показаний цьому пацієнту

для запобігання виразки?

а) сукральфат

б) гастроцепін

в) ранітидин

г) маалокс

д) місопростіл

029. При лікуванні Хелікобактеріоз найбільш ефективне застосування:

а) ранітідіна

б) оксациліну

в) де-нолу

г) де-нолу+ампіцилін (амоксицилін)

д) маалокса

030. Найбільш раціональний режим призначення антацидів

у хворих на виразкову хворобу:

а) за 20 хвилин до їди

б) відразу після їжі

в) через 20 хвилин після їжі та на ніч

г) через годину після їжі та на ніч

д) незалежно від їди 4-5 разів на день

031. Які гіпотензивні препарати

чи діють переважно як постгангліонарні адреноблокатори?

а) пентамін

б) клонідин

в) гуанетидину сульфат

г) анапрілін

д) хлорталідон

032. Які гіпотензивні препарати

чи діють на нейрогуморальні механізми регуляції АТ?

а) клонідин

б) каптоприл

в) міноксидил

г) гуанетидин

д) гідрохлортіазид

033. Вкажіть механізми гіпотензивної дії верошпірону:

а) зниження активності реніну плазми

б) блокада адренорецепторів

в) зменшення обсягу циркулюючої рідини

г) зниження загального периферичного опору

д) конкурентний антагоніст альдостерону

034. Біодоступність нифедипіну низька через:

а) пресистемної елімінації у печінці

б) низька абсорбція

в) зв'язування з білками плазми

г) інактивації у шлунково-кишковий тракт

035. Вкажіть тривалість гіпотензивної дії разової дози клофеліну

при вживанні:

а) 1-2 години

б) 6-8 годин

в) 10-12 годин

г) 2-24 години

д) до 3 діб

036. Купірування неускладненого гіпертонічного кризу слід починати:

а) із внутрішньом'язового введення 1,0 мл 0,01% розчину клофеліну

б) з 40 мг фуросеміду всередину

в) з 10-20 мг ніфедипіну сублінгвально

г) з 40 мг анаприліну всередину

д) із внутрішньовенного 1,0 мл 0,5% розчину фентоламіну

037. Каптоприл протипоказаний хворим:

а) з цирозом печінки

б) із хронічною нирковою недостатністю

в) із легеневим серцем

г) із цукровим діабетом

д) з виразковою хворобою

038. Вкажіть механізм дії нітрогліцерину:

а) блокада-адренорецепторів

б) спазмолітична, міотропна дія

на гладкі м'язи судинної стінки

в) блокада дрібних кальцієвих каналів клітинної мембрани

г) підвищення активності-рецепторів

судинної стінки коронарних артерій

д) підвищує повільний струм кальцію в клітину

039. Які протипоказання до призначення нітратів?

а) гострий інфаркт міокарда

б) артеріальна гіпертензія

в) гіпотонія

г) брадикардія

д) атріовентрикулярна блокада

040. Чим обумовлений антиангінальний ефект -адреноблокаторів?

а) розширення коронарних судин

б) зниження після переднавантаження на серце

в) зменшення роботи серця

г) центральний механізм дії

д) підвищення потреби міокарда у кисні

041. Вкажіть -адреноблокатор селективної дії:

а) образливий

б) тразикор

в) віскен

г) сектор

042. Яка біозасвоюваність пропранололу при пероральному прийомі?

043. Яка біодоступність верапамілу при пероральному прийомі?

044. Вкажіть протипоказання до призначення ніфедипіну:

а) артеріальна гіпертонія

б) серцева недостатність

в) бронхіальна астма

г) артеріальна гіпотонія

д) атріовентрикулярна блокада 2-го ступеня

045. Вкажіть препарат, що відноситься до 1-ї групи антиаритмічних засобів

(мембраностабілізуючої дії):

а) лідокаїн

б) ізоптин

в) кордарон

г) хінідин

д) дифінін

046. Вкажіть препарат, що відноситься до 2-ї групи антиаритмічних засобів

(Місцеві анестетики):

а) мекситіл

б) образливий

в) віскен

г) кордарон

д) новокаїнамід

047. Вкажіть препарат, що відноситься до 3-ї групи антиаритмічних засобів

(-адреноблокатори):

а) лідокаїн

б) тразикор

в) кордарон

г) хінідин

д) ізоптин

048. Вкажіть тривалість дії лідокаїну:

а) 20 хвилин

б) 60 хвилин

в) 1,5-2 години

д) 12 годин

049. Вкажіть період напіввиведення кордарону:

а) 4-6 годин

б) 1-2 години

в) 20-24 години

050. Вкажіть час досягнення максимальної концентрації ізоптину

у плазмі крові при прийомі внутрішньо:

а) 10 хвилин

б) 50 хвилин

в) 1,5-2 години

г) 8 годин

д) 10 годин

051. Вкажіть місцевий анестетик,

має найбільш виражену антиаритмічну дію:

а) лідокаїн

б) тримекаїн

в) ксикаїн

г) новокаїн

д) мекситіл

052. Розвиток толерантності до нітратів залежить, головним чином:

а) від шляху введення нітрату

б) від часу досягнення максимальної концентрації у крові

в) від тривалості дії

г) від комбінації з іншими ліками

д) жоден із перерахованих факторів

не впливає на розвиток толерантності

053. На відміну від ізосорбід-динітрату, 5-ізосорбід-мононітрат:

а) не піддається первинній пресистемній елімінації

при проходженні через печінку

б) не викликає розвитку толерантності

в) не викликає метгемоглобінії

г) не викликає головного болю

д) відмінності між препаратами відсутні

054. Назвіть протизапальний засіб пролонгованої дії:

а) аспірин

б) анальгін

в) піроксикам

г) індометацин

д) ортофен

055. При прийомі саліцилатів їхня найменша концентрація спостерігається:

а) у нирках

б) у печінці

в) у міокарді

г) у легенях

д) у головному мозку

056. Назвіть глюкокортикоїдні препарати з пролонгованою дією:

а) Преднізолон

б) полькортолон

в) дексаметазон

г) кеналог

д) метилпреднізолон

057. Вкажіть побічну дію нікотинової кислоти:

а) ліподистрофія

б) гіперурикемія

в) рабдоміоз

г) порушення зору

д) бронхоспазм

058. Вкажіть побічну дію секвестрантів жовчних кислот:

а) свербіж шкіри

б) діарея

в) запори

г) порушення зору

д) депресія

059. Який антигістамінний препарат протипоказаний

а) супрастін

б) піпольфен

в) димедрол

г) тавегіл

д) фенкарол

060. Який антигістамінний препарат протипоказаний

при лікуванні алергічних реакцій на введення ЛЗ,

а) піпольфен

б) супрастін

в) димедрол

г) тавегіл

д) фенкарол

061. Період напіввиведення ліків – це:

а) час досягнення максимальної концентрації ліків у плазмі

б) час, протягом якого ліки досягають системного кровотоку

в) час, протягом якого ліки розподіляються в організмі

г) час, за який концентрація ліків у плазмі знижується на 50%

д) час, за який половина введеної дози досягає органу-мішені

062. Терапевтичний індекс – це:

а) терапевтична доза ліків

б) відношення концентрації ліків в органі або тканині

до концентрації його у плазмі крові

в) діапазон між мінімальною та максимальною

г) відсоток не пов'язаних з білком ліків

д) діапазон між мінімальною та максимальною

терапевтичними концентраціями ліки

063. До рецепторних засобів конкурентної дії належать:

а) нестероїдні протизапальні засоби

б) -адреноблокатори

в) петлеві діуретики

г) нітрати

д) фторхінолони

064. При призначенні наступних лікарських засобів

слід враховувати функцію печінки та нирок:

а) ліпофільних, що утворюють неактивні метаболіти

б) ліпофільних, що утворюють активні метаболіти

в) гідрофільних

г) гепатотоксичних

д) нефротоксичних

065. Селективність дії лікарської речовини залежить:

а) від періоду напіввиведення

б) від способу прийому

в) від зв'язку з білком

г) від обсягу розподілу

д) від дози

066. Суворо дозозалежними є такі групи побічних ефектів:

а) фармацевтичні

б) токсичні

в) алергічні

г) мутагенні

д) синдром відміни

067. Перерахуйте групи препаратів із вузьким терапевтичним індексом:

а) -блокатори

б) пеніциліни

в) серцеві глікозиди

г) метилксантини

д) потужні діуретики

068. Препаратом вибору за наявності атипових збудників

(мікоплазма, хламідії) є:

а) еритроміцин

б) метронідазол

в) гентаміцин

г) карбеніцилін

д) цефуроксим

069. Препаратами вибору за наявності атипових збудників

(мікоплазма, хламідії) є:

а) макроліди

б) пеніциліни

в) аміноглікозиди

г) цефалоспорини

д) сульфаніламіди

070. Вкажіть антибактеріальний препарат,

що володіє найбільшою антианаеробною активністю:

а) еритроміцин

б) ампіцилін

в) тетрациклін

г) гентаміцин

д) цефотетан

071. Дисбактеріоз кишечника

викликають усі перелічені антибактеріальні препарати, крім:

а) напівсинтетичних препаратів

б) тетрациклінів

в) фторхінолонів

г) оральних цефалоспоринів

д) макроліди

072. Нефротоксичними є такі антибактеріальні препарати,

а) гентаміцину

б) карбеніциліну

в) еритроміцину

г) цефазоліну

д) ванкоміцину

073. Вкажіть антибактеріальний препарат,

не активний щодо пневмокока:

а) азитроміцин

б) пеніцилін

в) цефтріаксон

г) ципрофлоксацин

д) левоміцетин

074. Виберіть комбінацію антибактеріальних препаратів,

що володіє синергізмом дії та безпекою:

а) пеніциліни + тетрацикліни

б) пеніциліни + цефалоспорини

в) пеніциліни + макроліди

г) пеніциліни + аміноглікозиди

д) пеніциліни + сульфаніламіди

075. Добре проникають через гематоенцефалічний бар'єр

наступні антибактеріальні препарати:

а) пеніциліни

б) макроліди

в) тетрацикліни

г) аміноглікозиди

д) цефалоспорини

076. Препаратом вибору при крупозній пневмонії є:

а) цефаклор

б) доксициклін

в) метицилін

г) цефотаксим

д) пеніцилін

077. Препаратом вибору при фарингіті є:

а) цефаклор

б) тетрациклін

в) цефтазидим

г) офлоксацин

д) пеніцилін

078. Нове покоління макролідних антибіотиків

має наступні переваги:

а) висока біодоступність

б) широкий спектр антибактеріальної дії

в) бактерицидна дія

г) тривалий період напіввиведення

д) нирковий шлях екскреції

079. Фторхінолони відрізняються від хінолонів такими властивостями:

а) широким антибактеріальним спектром дії

б) бактеріостатичним дією

в) високою пенетрацією у тканині

г) постантибактеріальним ефектом

д) пероральним способом введення

080. Виберіть препарат,

максимально пригнічує секрецію соляної кислоти:

а) пірензепін

б) циметидин

в) карбеноксолон

г) антациди

д) омепразол

081. Максимальна кількість побічних ефектів серед Н2-блокаторів

викликає:

а) циметидин

б) роксатідин

в) нізатидин

г) ранітидин

д) фамотидин

082. Синтетичні аналоги простагландинів (енпростил, мізопростол)

викликають перелічені ефекти:

а) антисекреторну дію

б) секрецію барбітуратів

в) слизоутворення

г) репарантна дія

083. Пригнічує метаболізм інших препаратів:

а) омепразол

б) карбеноксолон

в) циметидин

г) фамотидин

д) гастроцепін

084. Тривалість антисекреторної дії омепразолу становить:

а) 2-4 години

б) 8-10 годин

в) 16-20 годин

д) 3 доби

085. Вкажіть препарат,

є потужним стимулятором слизутворення в шлунку:

а) карбеноксолон

б) платифілін

г) омепразол

д) метоклопрамід

086. За наявності ниркової недостатності

потрібна корекція доз препаратів:

а) аналогів простогландинів

б) омепразолу

в) Н2-блокаторів

г) сукральфату

д) холіноблокаторів

087. Вкажіть антисекреторний препарат, що блокує "протоновий насос"

а) метоклопрамід

б) карбеноксолон

в) пірензепін

г) сукральфат

д) омепразол

088. При патології нирок

виникають такі зміни фармакогенетики ліків:

а) порушення ниркової екскреції

б) збільшення концентрації ліків у плазмі крові

в) зменшення зв'язування з білками плазми

г) збільшення періоду напіввиведення

д) зменшення біодоступності

089. Цироз печінки викликає такі зміни фармакокінетики ліків:

а) зниження пресистемного метаболізму

б) зменшення зв'язування з білками плазми

в) збільшення періоду напіввиведення

г) збільшення біодоступності

д) зменшення обсягу розподілу

090. При серцевій недостатності

спостерігаються такі зміни фармакокінетики дигоксину:

а) зниження абсорбції у шлунково-кишковому тракті на 30%

б) зменшення зв'язування з білком плазми

в) посилення метаболізму у печінці

г) зниження ниркової екскреції

д) збільшення періоду напіввиведення

091. Алкоголь наводить:

а) до збільшення абсорбції ліків

в) до уповільнення метаболізму у печінці

г) до зниження ниркової екскреції

д) до збільшення періоду напіввиведення

092. Нікотин наводить:

а) до зменшення абсорбції ліків

б) до збільшення обсягу розподілу ліків

в) до збільшення зв'язку з білками плазми

г) до прискорення метаболізму у печінці

д) до посилення ниркової екскреції ліків

093. При стенокардії напруги показано:

а) ніфедипін

б) пропранолол

в) капотен

г) еналаприл

д) клонідин

094. При стенокардії Принцметалу (вазоспастичної) показаний:

а) ніфедипін

б) образливий

в) дипіридамол

г) допігить

д) каптоприл

095. Критерієм ефективності антиангінального засобу є:

а) приріст часу навантаження на ВЕМ >1 хвилини

б) збільшення кількості споживаного НТГ

в) приріст часу ВЕМ – проби >2 хвилин

г) зниження часу навантаження

д) перехід хворого з 2-го до 3-го функціонального класу стенокардії

096. До антиангінальних засобів відносяться:

а) курантил

б) капотен

в) аспірин

г) верапаміл

097. Для усунення нападу стенокардії застосовують:

а) сустак

б) нітронг

в) нітрогліцерин

г) верапаміл

д) дилтіазем

098. З антиангінальних засобів

при поєднанні ІХС та артеріальної гіпертонії показаний:

а) сустак

б) верапаміл

в) каптоприліл

г) курантил

д) еналаприл

099. Найімовірніше розвиток толерантності при застосуванні:

а) тринітролонгу

б) сустака

в) сублінгвального нітрогліцерину

г) ізосорбіту-5-мононітрату

д) нітронга

100. Методом контролю ефективності антиангінальної терапії є:

а) моніторування ЕКГ по Холтеру

б) контроль рівня ліпідів крові

в) добове моніторування АТ

г) вимірювання ФЗД (функції зовнішнього дихання)

д) вимірювання АТ в орто- та клиностазі

101. Препаратом вибору при стенокардії у хворого з брадикардією є:

а) піндолол

б) пропранолол

в) верапаміл

г) дилтіазем

д) метопролол

102. Препаратом вибору при стенокардії

у хворого із серцевою недостатністю є:

а) верапаміл

б) корінфар

в) дилтіазем

г) ацебуталол

д) нітросорбіт

103. До інгаляційних кортикостероїдів належать:

а) гідрокортизон

б) беклометазон

в) преднізолон

г) полькортолон

д) дексаметазон

104. До селективних 2-агоністів тривалої дії відноситься:

а) флутиказон

б) сальметерол

в) сальбутамол

г) фенатерол

д) тербуталін

105. Для усунення нападу бронхіальної астми застосовується:

а) іпратропія бромід

б) теопек

г) сальбутамол

д) задитий

106. Мембраностабілізатором для прийому внутрішньо є:

а) кетотифен

б) недокрамив натрію

в) хромоглікат натрію

г) супрастін

д) іпратропіум бромід

107. При синдромі "замикання" застосовують:

а) сальбутамол

б) фенотерол

в) теопек

г) еуфілін

д) адреналін

108. До муколітичних засобів відноситься:

а) кодеїн

б) хромоглікат натрію

в) ацетилцистеїн

г) сальметерол

д) теофілін

109. При одночасному застосуванні

підвищує концентрацію теофіліну в крові:

а) офлоксацин

б) пеніциліни

в) цефтріаксон

г) гентаміцин

д) бісептол

110. При одночасному застосуванні

знижує концентрацію теофіліну в крові:

а) пефлоксацин

б) циметидин

в) рифампіцин

г) еритроміцин

д) ампіокс

111. При підвищеному тиску в легеневій артерії

у хворого з бронхіальною астмою показаний:

а) верапаміл

б) ніфедипін

в) дігоксин

д) беклометазон

112. При бронхіальній астмі на фоні хронічного бронхіту показано:

а) іпратропія бромід

б) адреналін

в) ефедрин

г) кетотифен

д) супрастін

113. Вкажіть кращий шлях для введення лікарських препаратів

при застійній серцевій недостатності:

а) ректальний

б) сублінгвальний

в) усередину

г) внутрішньовенний

д) нашкірний

114. Перерахуйте препарати,

які мають прямим позитивним інотропним ефектом:

а) дігоксин

б) допамін

в) норадреналін

г) еуфілін

д) гідралазин

115. Вкажіть стан,

підвищують чутливість до серцевих глікозидів:

а) літній вік

б) тиреотоксикоз

в) легеневе серце

г) гіпокалемія

д) застійна серцева недостатність

116. Перерахуйте препарати, при взаємодії з якими

концентрація дигоксину в крові може підвищуватися:

а) фосфулагель

б) хінідин

в) верапаміл

г) аміодарон

117. Перерахуйте фактори, що уповільнюють всмоктування серцевих глікозидів

із шлунково-кишкового тракту:

а) хронічна ниркова недостатність

б) застійна серцева недостатність

в) виразка шлунка

г) спільний прийом із антацидними засобами

118. Перерахуйте фактори,

що забезпечують найбільшу безпеку та ефективність діуретиків

при тривалому лікуванні серцевої недостатності:

а) максимальні дози

б) середні дози

в) мінімальні дози

г) щоденний прийом

д) переривчастий прийом

119. Вкажіть найефективніший діуретик

для лікування застійної серцевої недостатності

з розвитком вторинного гіперальдостеронізму:

а) етакринова кислота (урегид)

б) хлорталідон (гігротон)

в) ацетазоламід (діакарб)

г) спіронолактон (верошпірон)

д) тріампур

120. Вкажіть основний терапевтичний ефект нітросорбіду

у хворих із серцевою недостатністю:

а) розширення переважно артеріол та зниження постнавантаження

б) розширення переважно венул та зниження переднавантаження

в) пряма позитивна інотропна дія

г) підвищення діурезу та зниження переднавантаження

121. Перерахуйте препарати, що мають холінолітичні побічні ефекти:

а) лідокаїн

б) хінідин

в) аміодарон (кордарон)

г) верапаміл

д) прокаїнамід (новокаїнамід)

122. Перерахуйте групи антиаритмічних засобів,

надають антифібриляторну дію:

а) серцеві глікозиди

б) антагоністи кальцію (4-я група)

в) -блокатори (2-я група)

г) аміодарон, бретилію тозилат (3-я група)

д) хінідин, прокаїнамід та інші препарати 1а групи

123. Перерахуйте препарати,

які можуть спровокувати напад передсердної тахіаритмії

при синдромі Вольфа-Паркінсона-Уайта:

а) дігоксин

б) аміодарон

в) верапаміл

г) пропранолол

д) етмозин

124. Вкажіть препарати, яким властивий аритмогенний ефект:

а) аймалін

б) мексилетин

в) пропафенон

г) аміодарон

125. Перерахуйте показання до лікування антиаритмічними препаратами:

а) аритмії, що спричиняють порушення гемодинаміки

б) суб'єктивна непереносимість аритмії

в) порушення ритму високих градацій

г) часті порушення ритму

126. Перерахуйте препарати,

основна антиаритмічна дія яких

пов'язано з подовженням атріовентрикулярної провідності:

а) пропранолол

б) лідокаїн

в) верапаміл

г) дігоксин

д) прокаїнамід

127. Вкажіть ефекти взаємодії аміодарону та дизопірамідів

(ритмілен, норпейс):

а) уповільнення метаболізму дизопіраміду

б) уповільнення метаболізму аміодарону

в) підвищення ризику розвитку побічних ефектів дизопіраміду

г) підвищення ризику розвитку побічних ефектів аміодарону

128. Вкажіть побічний ефект ніфідепіну:

а) брадикардія

б) бронхоспазм

в) набряки гомілок та стоп

г) розвиток F-V блокади

д) ульцерогенність

129. Вкажіть препарат,

механізмом гіпотензивної дії якого

є блокада-рецепторів:

а) клофелін

б) празозин

в) пропранолол

г) капотен

д) верапаміл

130. Препаратом вибору

у хворого з гіпертонією та застійною недостатністю кровообігу

є:

а) еналаприл

б) нітросорбід

в) клофелін

г) адельфан

д) пентамін

131. Оптимальний препарат для тривалої гіпотензивної терапії повинен:

а) впливати на обмін речовин

б) мати рикошетні реакції

в) мати синдром відміни

г) мати стабільну концентрацію у крові

д) викликати ортостатичні реакції

132. Перерахуйте групи гіпотензивних препаратів,

зменшують активність ренін-ангіотензин-альдостеронової системи:

а) інгібітори АПФ

б) -блокатори

в) центральні -агоністи

г) тіазидні діуретики

д) антагоністи кальцію

133. Вкажіть гіпотензивні препарати,

які необхідно з обережністю застосовувати

при поєднанні цукрового діабету та гіпертонічної хвороби:

а) верапаміл

б) пропранолол

в) дилтіазем

г) гіпотіазид

д) еналаприл

134. Проведення лікарського моніторингу

потрібно при лікуванні наступними групами препаратів:

а) протисудомними

б) 2-симпатоміметиками

в) метилксантинами

г) глюкокортикоїдами

д) М-холіноміметиками

135. Розвиток асистолії можливий при комбінації пропранололу:

а) із фенобарбіталом

б) з фуросемідом

в) з верапамілом

г) з фенітеїном

д) з циметидином

136. Ризик токсичних ефектів збільшується при комбінації гентаміцину:

а) з фуросемідом

б) з пеніциліном

в) із метилксантинами

г) з макролідами

д) з глюкокортикоїдами

КЛІНІЧНА ФАРМАКОЛОГІЯ

114 - а,б,в,г,д

115 - а,б,в,г,д

088 - а, б, в, г

089 - а, б, в, г

090 - а, б, г, д

124 - а,б,в,г,д

078 - а, б, в, г

079 - а,в,г,д

083 - а, б, в, г

МОДУЛЬ 3 КЛІНІЧНА ФАРМАКОДИНАМІКА. ПАЦІЄНТ І ЛІКИ

МОДУЛЬ 3 КЛІНІЧНА ФАРМАКОДИНАМІКА. ПАЦІЄНТ І ЛІКИ

Після освоєння теми студент має знати

1. Визначення фармакодинаміки.

2. Визначення антагоністів, агоністів, часткових агоністів.

3. Типи молекул-мішеней лікарських засобів (рецептори, ферменти, іонні канали).

4. Види фармакологічної відповіді: очікувана фармакологічна відповідь, гіперреактивність, тахіфілаксія, ідіосинкразія.

5. Принципи розробки програм контролю за ефективністю лікарських засобів.

6. Гострий фармакологічний тест (поняття, призначення, показання, правила проведення).

7. Методи оцінки впливу лікарських засобів на якість життя.

8. Методологію збирання фармакологічного анамнезу (поняття, клінічне значення, правила збирання, інтерпретація).

9. Прихильність хворого на лікування - комплієнтність (поняття, фактори, що впливають на прихильність до лікування, методи підвищення прихильності хворого на лікування).

10. Відповідальне самолікування.

Після освоєння теми студент має вміти

1. Інтерпретувати інформацію про фармакодинаміку лікарських засобів (з інструкції та ТКФЗ) для індивідуалізованого вибору лікарських засобів.

2. Розробляти програму контролю за ефективністю лікарських засобів з урахуванням їх фармакологічних ефектів.

3. Оцінювати вплив лікарських засобів на якість життя.

4. Проводити гострий фармакологічний тест, інтерпретувати його результати для вибору лікарських засобів.

5. Збирати та інтерпретувати фармакологічний анамнез.

6. Проводити заходи, що підвищують відданість хворого на медикаментозне лікування.

Література, необхідна освоєння теми

Основна

Кукес В.Г.Клінічна фармакологія – М.: Геотар-Медіа, 2008. – С. 80-94, 95-117.

Додаткова

Новик А.А., Іонова Т.І.Посібник із дослідження якості життя у медицині. – М.: Олма медіа груп, 2007. – 320 с.

Сергєєв П.В., Шимановський Н.Л., Петров В.І.Рецептори. – Волгоград, 1999. – 640 с.

Лекція, яку потрібно прослухати для освоєння теми

Кукес В.Г.«Введення у клінічну фармакологію. Фармакодинаміка».

http://lech.mma.ru/clinpharm/ucheb/pharm/lekt/1

Виконайте тестові завдання для самоконтролю

Виберіть одну або кілька відповідей.

I. Фармакодинаміка включає:

A. Процеси всмоктування, розподілу, метаболізму та виведення ЛЗ.

Б. Механізм дії, локалізацію дії та види дії ЛЗ.

B. Рух ЛЗ в організмі хворого. Г. Фізико-хімічні властивості ЛЗ.

ІІ. Встановіть відповідність термінів та їх визначень:

A. Агоністи.

Б. Антагоністи.

B. Часткові агоністи.

1. ЛЗ, що зв'язуються з тими самими рецепторами, що й ендогенні медіатори, що викликають «нульовий ефект».

2. ЛЗ, що зв'язуються з тими ж рецепторами, що й ендогенні медіатори, що викликають ефект рівний або більший за ефект даного медіатора.

3. ЛЗ, що зв'язуються з тими ж рецепторами, що і ендогенні медіатори, що викликають ефект більше нуля, але менше ефекту даного медіатора. ІІІ. Встановіть відповідність термінів та їх визначень:

A. Ідіосинкразія. Б. Тахіфілаксія.

B. Толерантність.

1. Зниження терапевтичного ефекту, що спостерігається при повторному застосуванні ЛЗ.

2. Генетично обумовлена ​​збочена реакція на певний лікарський препарат, що виявляється підвищеною чутливістю до нього та/або тривалим ефектом та пов'язана з генетично детермінованим дефектом ферментних систем.

3. Зниження терапевтичного ефекту, що спостерігається при тривалому (багаторазовому) застосуванні ЛЗ.

Завдання 3.1. Наведіть приклади лікарських засобів – агоністів та антагоністів різних типів (а точніше класів) рецепторів, заповнивши схему на рис. 3.1. У центрі прямокутниках вкажіть рецептори, зверху і знизу в стрілках - приклади лікарських засобів, які є їх агоністами чи антагоністами.

Докладніша інформація про взаємодію ЛЗ з молекулами-мішенями в лекції R. Leurs «Drug-receptor interaction» (англійською мовою): www.ux1.eiu.edu/~cfthb/classes/4790/pdfs/ Drug-Receptor_Interactions.pdf

Навіщо лікарю потрібна інформація про фармакодинаміку лікарських засобів та як її інтерпретувати

Фармакодинаміка - це розділ клінічної фармакології, що вивчає механізм дії, локалізацію дії та види дії ЛЗ у хворого. У спрощеному вигляді фармакодинаміка дозволяє зрозуміти, що ЛЗ робить із організмом хворого. При цьому виділяють основний або терапевтичний ефект ЛЗ – фармакологічний.

Мал. 3.1.Лікарські засоби - агоністи та антагоністи різних рецепторів

ефект, який спрямований на досягнення поставленої мети лікування. Всі фармакологічні ефекти, що не належать до основного, розцінюються як побічні, які можуть бути не лише «негативними» для здоров'я хворого (небажані побічні реакції), а й «позитивними». Інформація про фармакодинаміку ЛЗ міститься в розділі «Фармакологічна дія» інструкції та ТКФЗ та призначена для лікаря. На підставі даної інформації у лікаря формується уявлення про значення фармакодинамічних ефектів у розвитку терапевтичного та небажаних лікарських реакцій (табл. 3.1), що визначає показання та протипоказання до застосування ЛЗ, а отже, і вибір даного ЛЗ. Таким чином, лікар повинен розуміти клінічне значення викладеної інформації для індивідуалізованого вибору як групи ЛЗ, так і окремих ЛЗ (у тому числі з урахуванням результатів гострого фармакологічного тесту), розробки програми контролю ефективності лікування (у тому числі оцінки динаміки якості життя хворого на фоні застосування ЛЗ).

Таблиця 3.1. Значення інформації про фармакодинаміку бісопрололу для розуміння його терапевтичних

та несприятливих побічних реакцій

Лікарський засіб

Молекула-мішень та характер взаємодії з нею

(Агоніст або антагоніст)

Локалізація дії

Види дії

Фармакодинамічний ефект

Терапевтичний ефект та відповідне показання до застосування

Несприятлива побічна реакція

Бісопролол

β1-адренорецептори

Кардіоміоцити

Негативний іно-, дромо-, хроно-, батмотропний. Зниження потреби міокарда у кисні.

Гіпотензивний

Антиангінальний – ІХС. Зниження ЧСС, антиаритмічний ефект – тахіаритмії; ХСН.

Гіпотензивний -

артеріальна

гіпертензія

брадикардія. Пригнічення АВ-провідності

У великих

β 2 -адренорецеп-

Підшлункова залоза, скелетні м'язи, гладка мускулатура периферичних артерій, бронхів та матки

Зниження тонусу бронхів; гіперглікемічний ефект; зниження периферичного кровообігу

Не використовується

Бронхоспазм Порушення вуглеводного обміну, периферичного кровотоку

Виконайте завдання для самостійної роботи

Завдання 3.2. Опишіть фармакодинаміку запропонованого викладачем ЛЗ чи ЛЗ за своєю майбутньою спеціальністю, заповнивши таблицю за аналогією з табл. 3.1. Використовуйте розділ «Фармакологічна дія» ТКФБ.

Як знання особливостей фармакодинаміки допомагає лікареві вибрати конкретний лікарський засіб із групи?

Знання особливостей фармакодинаміки ЛЗ лежить в основі вибору конкретного ЛЗ усередині обраної групи відповідно до принципу збігу профілю ЛЗ (особливості фармакодинаміки ЛЗ з розділу «Фармакологічна дія» інструкції та ТКФЗ) та профілю пацієнта (індивідуальні особливості пацієнта). Як приклад представлений вибір ЛЗ із групи β-адреноблокаторів для лікування артеріальної гіпертензії на основі особливостей фармакодинаміки (табл. 3.2).

Таблиця 3.2.Вибір лікарського засобу із групи β-адреноблокаторів для лікування артеріальної гіпертензії на основі особливостей фармакодинаміки

Особливість фармакодинаміки

Принципи вибору

Пропранолол Карведилол

Блокада β Γ

та β2-адренорецепторів

Вибираємо при супутній портальній гіпертензії. Не вибираємо при супутній ХОЗЛ, цукровому діабеті, гіперліпідемії, у хворих молодого та середнього віку (сексуально-активних)

Атенолол Метопролол Бісопролол Небіволол

Селективна блокада βl-адренорецепторів

Вибираємо при супутній ХОЗЛ (особливо бісопролол та небіволол), у хворих молодого та середнього віку (сексуальноактивних) (особливо бісопролол та небіволол), цукровому діабеті у стадії компенсації (при добре підібраній цукрознижувальній терапії)

Закінчення табл. 3.2

Особливість фармакодинаміки

Принципи вибору

Карведилол Небіволол

Наявність додаткових вазодилатирующих властивостей: у небівололу за рахунок здатності стимулювати вивільнення оксиду азоту в ендотелії судин, а у карведилолу за рахунок α-адреноблокуючого ефекту

Вибираємо при супутньому облітеруючому атеросклерозі судин нижніх кінцівок (за винятком випадків критичної ішемії)

Карведилол

Метопролол

(у вигляді сук-

Бісопролол

Небіволол

Доведений у рандомізованих дослідженнях позитивний вплив на прогноз у хворих з хронічною серцевою недостатністю за рахунок зниження активності симпатикоадреналової системи

Вибираємо при супутній хронічній серцевій недостатності у стадії компенсації (за відсутності ознак затримки рідини). При цьому небіволол вибираємо тільки хворим з хронічною серцевою недостатністю віком від 75 років (є докази ефективності цього ЛЗ тільки для цієї категорії пацієнтів). Слід віддавати перевагу карведилолу при поєднанні хронічної серцевої недостатності та артеріальної гіпертензії III ступеня (карведилол має більш виражений антигіпертензивний ефект за рахунок α-адреноблокуючого ефекту)

Виконайте завдання для самостійної роботи

Завдання 3.3. Використовуючи розділ «Фармакологічна дія» ТКФС, розробіть принципи вибору ЛЗ усередині групи ЛЗ на тему практичного заняття (група ЛЗ пропонується викладачем)

або з вашої майбутньої спеціальності. Для цього заповніть таблицю за аналогією до табл. 3.2.

Вибір ЛЗ із групи повинен здійснюватися не тільки на основі особливостей фармакодинаміки, а й особливостей фармакокінетики, облік яких також сприяє вибору індивідуалізованого режиму дозування (див. модуль 2).

Як розробляти індивідуальну програму контролю ефективності лікарського засобу?

У процесі застосування ЛП лікар має здійснювати контроль ефективності, тобто. за ступенем досягнення поставлених цілей лікування (первинних та вторинних). Для цього необхідно розробити програму контролю ефективності, яка буде методами (клінічні, лабораторні та інструментальні), що оцінюють розвиток терапевтичних ефектів ЛП, а також терміни їх виконання та рівень зміни результатів цих методів на фоні лікування (табл. 3.3). Вибрані методи у програмі контролю ефективності ЛП повинні давати інформацію лікареві про ступінь досягнення первинних та вторинних цілей лікування у хворого залежно від показань до застосування ЛП. На основі результатів лікар приймає відповідні рішення: коригує дози ЛП, продовжує застосування ЛП, скасовує ЛП, приєднує інші ЛП. Ці рішення лікар приймає також відповідно до результатів програми контролю за безпекою (див. модуль 4). Програми контролю ефективності та безпеки реалізуються паралельно та взаємно доповнюють одна одну. Для розробки програми контролю ефективності ЛП слід використовувати інформацію з розділів «Фармакологічна дія» та «Покази» інструкції та ТКФЗ.

Виконайте завдання для самостійної роботи

Завдання 3.4. Розробте програму контролю ефективності застосування ЛЗ, запропонованого викладачем, або самостійно обравши ЛЗ за своєю майбутньою спеціальністю. Заповніть таблицю за аналогією до табл. 3.3. Використовуйте розділ «Фармакологічна дія» ТКФБ.

Таблиця 3.3.Методи контролю ефективності лікування хворого із загостренням виразкової хвороби дванадцятипалої кишки

Показання у пацієнта

Лікарський засіб

Методи оцінки ефективності

Загоєння виразкового дефекту

Лікарський засіб 1 - рабепразол

Механізм 1 . Інактивує сульфгідрильні групи Н+/К+-АТФази. Блокує заключну стадію секреції HCl, знижуючи вміст базальної та стимульованої секреції. Механізм 2. Проникає в парієтальні клітини шлунка і концентрується в них, надаючи цитопротекторну дію та збільшуючи секрецію гідрокарбонату.

Клінічні методи: зменшення больового синдрому або його усунення, усунення печії, відрижки. Лабораторні методи: лабораторні маркери ерадикації Н. pylory(при сумісному застосуванні з антибіотиками) – див. нижче. Інструментальні методи: ЕГДС – рубцювання виразки, зменшення її розміру. рН-метрія - підвищення внутрішньошлункового рН

Ерадикація Н. pylori

Лікарський засіб 1 – рабепразол

Механізм 1. Збільшує концентрацію антибактеріальних засобів у слизовій оболонці шлунка.

Механізм 2. Підвищує рН, збільшуючи активність антибіотиків (амоксициліну, кларитроміцину). Механізм 3. Антихелікобактерний ефект (пригнічують ріст Н. pylori in vivo,діючи на систему АТФази бактерій)

Клінічні методи: відсутні. Лабораторні методи: уреазний дихальний тест через 3-4 тижні після закінчення терапії.

Інструментальні методи:

гістологічне дослідження біоптату;

уреазна проба з біоптатом

Закінчення табл.3.3

Показання у пацієнта

Лікарський засіб

Механізм дії лікарського засобу, що відповідає показанню

Методи оцінки ефективності

Кларитроміцин

Механізм 1. Пригнічує синтез білка шляхом оборотного зв'язування з 50S-субодиницею рибосом Н. pylori,маючи бактеріостатичний ефект

Амоксицилін

Механізм 1. Порушують утворення клітинної стінки бактерій за рахунок незворотного зв'язування з пеніцилінозв'язуючими білками, надаючи бактерицидний ефект.

Як використовувати оцінку якості життя як клінічний метод контролю ефективності лікарського препарату?

Важливим клінічним методом контролю ефективності ЛП вважається оцінка якості життя динаміці. Якість життя - це інтегральний показник, що включає кілька компонентів:

Функціональний стан (працездатність, толерантність до фізичного навантаження, виконання домашньої роботи);

Симптоми, пов'язані із захворюванням або його лікуванням (біль, задишка, побічні ефекти ЛЗ);

Психічний стан (депресія або збудження, які можуть бути як наслідком захворювання, так і застосування ЛЗ);

Соціальна активність;

Статева функція;

Задоволення медичною допомогою.

Чинники, що погіршують якість життя, можна розділити на дві групи:

Фактори, пов'язані із захворюванням (симптоми захворювання, що заважають жити);

Чинники, пов'язані з медикаментозною терапією (незручність застосування ЛП, розвиток небажаних лікарських реакцій).

Ступінь вираженості факторів, пов'язаних із захворюваннями, оцінюють за допомогою опитувальників. При цьому існують універсальні опитувальники для оцінки якості життя у хворих із будь-якими захворюваннями, проте вони, як правило, дуже громіздкі, їх заповнення та інтерпретація займають багато часу у пацієнтів та лікарів, тому розроблені спеціальні опитувальники для хворих на найбільш поширені захворювання. Хворий повинен самостійно відповісти на запитання опитувальника, а лікар підраховує загальну кількість балів відповідно до відповідей пацієнта. Динаміка суми балів на фоні лікування ЛП та буде методом контролю ефективності ЛП.

Універсальний опитувальник якості життя EQ-5D (російською мовою): http://forum.disser.ru

Докладніша інформація про оцінку якості життя знаходиться в статті: М'ясоїдова Н.А., Тхостова Е.Б., Білоусов Ю.Б.Про оцінку якості життя при різних серцево-судинних захворюваннях: http://www.trimm.ru/php/content.php?group=2&id=3726

Аналіз факторів, пов'язаних із медикаментозною терапією, дозволяє провести корекцію застосування ЛП у процесі лікування.

Наприклад, проаналізуємо якість життя пацієнта 53 років із діагнозом: ІХС; постінфарктний кардіосклероз. Артеріальна гіпертензія 2-го ступеня, ІІІ стадії, дуже високого ризику. ХСН ΙΙ ФК за NYHA. Пацієнт отримує еналаприл 5 мг 2 рази на добу, бісопролол 2,5 мг 1 раз на добу, фуросемід 40 мг 2 рази на тиждень, спіронолактон 25 мг 1 раз на добу. Пацієнт скаржиться на задишку при ходьбі 300-500 м, підйомі на 4 поверх (живе на 5 поверсі в будинку без ліфта), приблизно 1 раз на тиждень виникає нічна задишка; неможливість займатися фізичною працею та спортом

(раніше працював автомеханіком, захоплювався лижним спортом); у день прийому фуросеміду змушений залишатися вдома через діуретичний ефект, тому змушений перейти на менш оплачувану роботу зі змінним графіком; також відзначає невеликий сухий кашель (від призначення групи блокаторів ангіотензинових рецепторів відмовився через їхню вищу вартість). Чинники, що погіршують якість життя хворого, представлені у табл. 3.4.

Таблиця 3.4.Чинники, що погіршують якість життя хворого на хронічну

серцевою недостатністю

Оцінка факторів, що погіршують якість життя, пов'язаних із захворюваннями у даного пацієнта, проводиться за допомогою спеціального опитувальника для хворих на хронічну серцеву недостатність, так званий Мінессотський опитувальник (табл. 3.5). Зниження суми балів за цим опитувальником на фоні лікування переліченими ЛП свідчить про ефективність терапії. Вплив на якість життя виявлених факторів, пов'язаних із медикаментозним лікуванням, у даному випадку можна зменшити шляхом корекції терапії:

Замінити прийом фуросеміду 2 рази на тиждень на щоденний прийом гідрохлортіазиду у дозі 25 мг на добу;

Замінити інігібітор АПФ еналаприл на антагоніст рецепторів ангіотензинових лозартан, який набагато рідше викликає сухий кашель, але аналогічний за ефективністю;

Призначити комбінований препарат, що містить в одній таблетці лозартан та гідрохлортіазид у відповідних дозах.

Таблиця 3.5.Мінессотський опитувальник оцінки якості життя хворих з хронічною серцевою недостатністю

Чи заважала вам серцева недостатність жити так, як хотілося б протягом останнього місяця через:

1. Набряків гомілок, стоп

0, 1, 2, 3, 4, 5

2. Необхідності відпочивати вдень

0, 1, 2, 3, 4, 5

3. Проблеми підйому сходами

0, 1, 2, 3, 4, 5

4. Труднощі працювати по дому

0, 1, 2, 3, 4, 5

5. Проблеми з поїздками поза домом

0, 1, 2, 3, 4, 5

6. Порушень нічного сну

0, 1, 2, 3, 4, 5

7. Проблеми спілкування з друзями

0, 1, 2, 3, 4, 5

8. Зниження заробітку

0, 1, 2, 3, 4, 5

9. Неможливості займатися спортом, хобі

0, 1, 2, 3, 4, 5

10. Сексуальні порушення

0, 1, 2, 3, 4, 5

11. Обмежень у дієті

0, 1, 2, 3, 4, 5

12. Почуття нестачі повітря

0, 1, 2, 3, 4, 5

13. Необхідності лежати у лікарні

0, 1, 2, 3, 4, 5

14. Почуття слабкості, млявості

0, 1, 2, 3, 4, 5

15. Необхідність платити

0, 1, 2, 3, 4, 5

16. Побічної дії антибіотиків

0, 1, 2, 3, 4, 5

17. Почуття тягаря для рідних

0, 1, 2, 3, 4, 5

18. Почуття втрати контролю

0, 1, 2, 3, 4, 5

19. Почуття занепокоєння

0, 1, 2, 3, 4, 5

20. Погіршення уваги, пам'яті

0, 1, 2, 3, 4, 5

21. Почуття депресії

0, 1, 2, 3, 4, 5

Варіанти відповідей: 0 – ні; 1 - дуже мало... 5 - дуже багато (найвища якість життя - 0 балів; найнижча - 105 балів).

Виконайте завдання для самостійної роботи

Завдання 3.5. Перерахуйте фактори, що визначають якість життя у курованого хворого, заповнивши таблицю, аналогічну табл. 3.5.

Виконайте завдання для самостійної роботи

Завдання 3.6. Знайдіть або розробте самостійно опитувальник оцінки якості життя хворих із захворюванням за вашою майбутньою спеціальністю. Використовуйте як приклад опитувальник для оцінки якості життя хворих на хронічну серцеву недостатність (Мінессотський опитувальник; див. табл. 3.5).

Що таке гострий фармакологічний тест та як інтерпретувати його результати для індивідуалізованого вибору лікарських засобів?

Гострий фармакологічний тест - це аналіз одноразового застосування ЛЗ у хворого, що курується, що дозволяє прогнозувати терапевтичний ефект і розвиток небажаних лікарських реакцій при застосуванні ЛП.

У клінічній практиці широко застосовуються діагностичні фармакологічні тести: проба з добутаміном для виявлення ішемії міокарда, проба з АКТГ та дексаметазоном при діагностиці надниркової недостатності, проба з неостигміном при діагностиці міастенії, діазепамова проба в діагностиці аффективів; У ряді випадків фармакологічний тест допомагає не тільки підтвердити діагноз, але й визначити потенційну ефективність лікування, наприклад, очевидно, що при позитивному тесті з сальбутамолом бронхолітики групи β2~адреноміметиків будуть ефективні.

Поняття гострий фармакологічний тест має на увазі оцінку ефективності та безпеки ЛЗ за відомого діагнозу. При цьому використовуються вибрані методи контролю ефективності та безпеки. Для проведення гострого фармакологічного тесту лікарю потрібна інформація про фармакодинаміку ЛЗ із розділу «Фармакологічна дія» та про фармакокінетику ЛЗ із відповідного розділу інструкції з медичного застосування.

ЛЗ чи ТКФС.

Правила проведення гострого фармакологічного тесту розберемо з прикладу. Лікар-терапевт викликаний на консультацію у неврологічне відділення до пацієнта 70 років із діагнозом: атеросклероз судин головного мозку. Атеросклеротичний стеноз внутрішніх сонних артерій. Дисциркуляторна енцефалопатія II стадії у зв'язку з підвищенням артеріального тиску до 175/100 мм рт.ст., супрово-

очікується помірним головним болем. З анамнезу відомо, що у пацієнта двічі за останні 7 років спостерігалося підвищення артеріального тиску до 160/100 мм рт.ст., звичний артеріальний тиск становить 140/80, гіпотензивна терапія раніше не проводилася. Вихідна ЧСС – 86 за хв. Яким чином необхідно проводити вибір гіпотензивної терапії за допомогою гострого фармакологічного тесту? В даному випадку пацієнт має високий ризик розвитку порушення мозкового кровообігу при надмірному зниженні АТ, отже, з боку лікаря потрібен посилений контроль безпеки терапії, що проводиться, доречно проведення гострого фармакологічного тесту.

Пацієнту було призначено бісопролол у дозі 2,5 мг.

Проведено контроль АТ та ЧСС:

Через 1 год – 170/95 мм рт. ст; 70 хв;

Через 2 год – 160/90 мм рт.ст.; 68 хв;

Через 3 год – 140/90 мм рт.ст.; 66 хв;

Через 4 год – 110/70 мм рт.ст.; 55 хв;

Через 6 год – 115/70 мм рт.ст.; 57 хв;

Через 12 год – 160/70 мм рт.ст.; 58 хв.

Згідно з ТКФС, максимальна концентрація бісопрололу досягається через 2-4 години після прийому. В даному випадку через 4 та 6 годин у пацієнта спостерігалося надмірне зниження артеріального тиску, а через 12 годин терапія була недостатньою. Отже, початкову дозу бісопрололу слід зменшити, а через 8-10 год. виконати додатковий прийом препарату.

Що таке фармакологічний анамнез та як здійснювати його збір?

Збір фармакологічного анамнезу – обов'язкова процедура для лікаря. Фармакологічний анамнез - сукупність відомостей про раніше вживані хворі лікарські засоби, способи їх застосування, дози, ефективність, небажані побічні реакції, ознаки непереносимості, лікарської залежності. Фармакологічний анамнез дозволяє виявити ефективні раніше ЛЗ та/або ЛЗ, що викликали розвиток небажаних лікарських реакцій. Це може визначати вибір як груп, так і конкретних ЛЗ та їх доз. Для цього лікарю необхідно:

Виявити, які ЛЗ пацієнт не переносить;

Визначити, як конкретно виявлялася непереносимість лікарських засобів;

При цьому необхідно звернути увагу, що пацієнти часто по-своєму трактують питання лікаря, наприклад, під алергією на нітрогліцерин багато пацієнтів розуміють головний біль;

Встановити причину непереносимості:

Справжню алергічну реакцію та ступінь її тяжкості. Наприклад, при нетяжкій алергічній реакції на пеніциліни (кропив'янка) можливе призначення β-лактамних антибіотиків із групи цефалоспоринів, а при набряку Квінку або анафілактичному шоці всі препарати з цього класу протипоказані;

Розвиток тяжких небажаних реакцій, при цьому необхідно з'ясувати правильність режиму дозування та прийому препарату, врахувати можливі взаємодії. Приклад - глікозидна інтоксикація при прийомі дигоксину в добовій дозі 0,25 мг разом із фуросемідом;

Розвиток легких, але порушують якість життя пацієнта НПР, наприклад кашель прийому інгібіторів АПФ;

Неправильний режим дозування та/або прийому ЛЗ. Наприклад, надто висока доза гіпотензивного ЛЗ може призвести до ортостатичного колапсу; розжовування ретардованих форм ніфедипіну може супроводжуватися вираженою тахікардією, слабкістю, припливами;

Оцінити ефективність терапії, що проводиться раніше, і виявити можливі причини неефективності:

Нерегулярний прийом препарату (слід звернути увагу на те, що деякі пацієнти, говорячи про те, що приймають ліки постійно, мають на увазі тоді, коли почуваються погано);

Недостатня доза/кратність прийому препарату. Наприклад, прийом амоксициліну/клавуланової кислоти у дозі 625/125 мг 2 рази на добу не забезпечує необхідного антибактеріального ефекту;

Неправильна техніка введення ЛЗ. Наприклад, інгаляційні бронхолітичні препарати можуть бути неефективними при неправильному виконанні інгаляції препарату, відповідно замість збільшення дози необхідно навчити пацієнта;

Розвиток толерантності. При розвитку толерантності до нітратів необхідно забезпечити безнітратний проміжок, а не збільшувати дозу препарату;

Наприклад, застосування клопідогрелу спільно з інгібіторами протонного насоса знижує антиагрегаційний ефект першого;

Тяжкий перебіг захворювання. Неефективність нітратів може говорити про розвиток нестабільної стенокардії;

Неправильний діагноз. Наприклад, нітрогліцерин майже неефективний при кардіалгіях некоронарного генезу;

Синдром відміни. Гіпертонічний криз, що виник унаслідок скасування клонідину, як правило, не усувається стандартними засобами і вимагає призначення клонідину.

Іноді збирання фармакологічного анамнезу сприяє уточненню діагнозу. Наприклад, інтерстиціальний фіброз легень може бути викликаний прийомом аміодарону. Особливу важливість фармакологічний анамнез має при загостреннях хронічних захворювань, наприклад, гіпоглікемічна кома може бути викликана відносною або абсолютною передозуванням гіпоглікемічних препаратів.

Виконайте завдання для самостійної роботи

Завдання 3.7. Сформулюйте питання для збору фармакологічного анамнезу, відповідні необхідної інформації, у хворого із захворюванням за вашою майбутньою спеціальністю, за іншої клінічної ситуації, запропонованої викладачем, або хворого, що курується. Заповніть таблицю. 3.6.

Таблиця 3.6.Фармакологічний анамнез

Закінчення табл. 3.6

Необхідна для фармакологічного анамнезу інформація

Сформульоване питання

Отримана від хворого інформація

Ефективність застосовуваних нині лікарських засобів

Переносність (безпека) лікарських засобів, що застосовуються в даний час

Лікарські засоби, що застосовувалися раніше в аналогічних ситуаціях

Причини припинення прийому застосовуваних раніше лікарських засобів

Інші лікарські засоби, що застосовуються при супутніх захворюваннях або інших цілях (оральні контрацептиви)

Супутня терапія «альтернативними» засобами: фітопрепаратами, гомеопатичними препаратами

Небажані лікарські реакції при прийомі раніше застосовуваних лікарських засобів

Ставлення до алкоголю, куріння, наркотиків

Що таке комплієнтність (прихильність), і навіщо лікареві потрібно її покращувати?

Комплієнтність – це прихильність пацієнта до лікування. Від комплаентності може залежати ефективність і безпека ЛП, що призначаються пацієнту. Жоден сучасний та ефективний метод лікування не буде достатньо ефективний у разі, якщо пацієнт не

Розуміє, навіщо йому це потрібно, та не виконує розпоряджень. Слід пам'ятати, що уявлення пацієнта про свою хворобу та необхідне лікування формується не тільки зі слів лікаря, а й від знайомих, засобів масової інформації, реклами. Найчастіше переконати пацієнта дотримуватися лікарських рекомендацій непросто. Крім того, потрібно врахувати, що молодому лікарю іноді складніше вплинути на думку старших за віком та соціальним станом пацієнтів. Нижче наведено деякі можливі причини недотримання пацієнтом лікарських рекомендацій: небажання приймати ліки (боїться «вживати хімію», прихильність до народної медицини, екстрасенсів та ін.):

Пацієнт може побоюватися небажаних побічних реакцій (навіть раніше невідомих);

Висока вартість лікування;

Незручність, пов'язана з лікуванням (наприклад, необхідність відмови від керування автомобілем при прийомі транквілізаторів, виражений сечогінний ефект при прийомі діуретиків, необхідність частого контролю коагулограми при прийомі антикоагулянтів);

Одночасний прийом великої кількості таблеток;

Тривалість лікування, особливо коли пацієнт не відчуває настання ефекту (наприклад, багато пацієнтів, які страждають на артеріальну гіпертензію, часто запитують: «Навіщо мені все життя приймати стільки ліків, якщо я не відчуваю свого тиску?») або коли небажана побічна реакція більш « значуща» для пацієнта, ніж терапевтичний ефект ЛЗ.

Виконайте завдання для самостійної роботи

Завдання 3.8. Проаналізуйте випадок із клінічної практики. При відповіді на запитання використовуйте інформацію з ТКФБ «Бісопролол» (розділи «Фармакокінетика», «Режим дозування») Державного реєстру лікарських засобів (www.regmed.ru).

Пацієнтці П. 82 років щодо артеріальної гіпертензії дільничним терапевтом призначено бісопролол у дозі 5 мг 1 раз на добу. При черговому візиті до лікаря через високі показники АТ рекомендовано збільшити дозу бісопрололу до 10 мг на добу. Однак в аптеці, куди звернулася пацієнтка, не було таблеток бісопрололу по 5 мг, у зв'язку з чим вона придбала таблетки по 2,5 мг. Пацієнтка почала приймати бісопролол по 2,5 мг 4 рази на добу,

пояснюючи це згодом незручністю одномоментного прийому одразу 4 таблеток. Через 5 днів прийому бісопрололу в такому режимі у хворої розвивалася картина гіпертонічного кризу, у зв'язку з чим вона госпіталізована до стаціонару.

1. Яка найімовірніша причина розвитку гіпертонічного кризу у хворої, з фармакокінетики бісопрололу?

2. Які причини низької комплаентності у хворої?

Для забезпечення максимальної комплаентності перед призначенням ЛЗ пацієнтові лікар повинен:

Вирішити, чи зможе пацієнт з об'єктивних причин дотримуватись плану лікування і чи можливо забезпечити регулярний контроль терапії (наприклад, візити до поліклініки для контролю МНО при призначенні варфарину можуть бути скрутними для літніх пацієнтів, також літні пацієнти можуть забувати приймати ЛП тощо). ;

По можливості призначити ЛП 1-2 рази на день;

По можливості призначати комбіновані ЛП;

Призначати зручні засоби доставки ЛЗ (наприклад, інгалятори);

Не міняти без необхідності один ЛП на інший і не заплутувати пацієнта різноманітними назвами препаратів, використовуйте МНН;

Надати чітку та зрозумілу інформацію щодо застосування

ЛП (табл. 3.7).

Таблиця 3.7.Інформація, яку повинен надати лікар пацієнту, при-

варальний оральний антикоагулянт варфарин

Продовження табл. 3.7

Як і коли застосовувати лікарський засіб

Варфарин приймається всією добовою дозою одночасно, бажано о 17-19 год. Запивають таблетки водою. Не рекомендується прийом разом із їжею, можна приймати натще.

Щоб уникнути розвитку ускладнень, необхідна доза варфарину контролюється аналізом крові, який називають МНО. Іноді у відповідях лабораторії може позначатися INR. Протягом усього періоду прийому варфарину МНО має бути в інтервалі 2,0-3,0. Якщо МНО менше 2,0, то згортання крові не знижено і можливі тромботичні ускладнення. Якщо МНО більше 4,0 – дуже реальні геморагічні ускладнення. Підвищення МНВ від 2,5 до 4,0 говорить про необхідність зниження дози препарату, але зазвичай не несе прямої загрози. При деяких захворюваннях необхідним верхнім кордоном МНО вважається 4,0-4,4.

Якщо немає можливості визначати МНО, допустимо контроль за протромбіновим часом (ПВ), проте цей спосіб значно менш надійний. Для розрахунку дози варфарину інші аналізи крові не потрібні. Для виявлення побічної дії препарату періодично призначаються загальний аналіз крові, сечі та деякі біохімічні дослідження. Добір дози варфарину.

Найбільш складний та відповідальний етап. Період підбору в середньому займає від 1 до 2 тижнів, але в деяких випадках збільшується до 2 місяців. У цей час вам знадобиться часте визначення МНО до 2-3 разів на тиждень або щодня. Щоразу, отримавши черговий результат дослідження, ваш лікар визначить зміну дози ліків та дату чергового аналізу. Якщо в кількох аналізах поспіль МНО залишається в інтервалі 2,0-2,5, це означає, що дозу варфарину підібрано. Подальший контроль лікування буде значно простішим.

Продовження табл. 3.7

Запитання, на які необхідно дати відповідь хворому

Конкретна інформація для хворого щодо певного лікарського засобу

Контроль дози варфарину.

Якщо дозу препарату підібрано, достатній рідкісний контроль – спочатку 1 раз на 2 тижні, потім 1 раз на місяць. Окремо визначається періодичність додаткових досліджень. Необхідність позачергового визначення МНО може виникнути у низці випадків, про які ми поговоримо нижче. За будь-яких сумнівів зверніться до свого лікаря за порадою

Як довго слід застосовувати лікарський засіб

Терапія варфарином тривала, іноді довічна. Для визначення тривалості терапії в деяких випадках потрібне проведення генетичних досліджень

Коли і які позитивні ефекти лікарського засобу можна очікувати

Лікарський препарат покликаний захистити ваше життя, не допускаючи розвитку інфарктів, інсультів, тромбоемболій

Можливі проблеми, які можуть виникнути при застосуванні лікарського засобу та що робити при виникненні цих проблем

Застосування варфарину може спричинити розвиток кровотеч.

Негайно повідомте лікаря, якщо ви виявили:

Чорний (дьогтеподібний) колір стільця;

Рожевий чи червоний колір сечі;

Кровотечі з носа або ясен (у тому числі при чищенні зубів);

Надзвичайно рясні або тривалі виділення при менструації;

Синці або припухлості на тілі, що виникають без видимої причини;

Будь-які значні зміни у самопочутті та здоров'ї;

Поява шкірних плям на стегнах, черевній стінці, молочних залозах

Продовження табл. 3.7

Запитання, на які необхідно дати відповідь хворому

Конкретна інформація для хворого щодо певного лікарського засобу

Які харчові продукти, напої (у тому числі алкогольні) та інші лікарські засоби (у тому числі фітопрепарати) слід уникати

Що потрібно уникати:

застосування інших препаратів, що впливають на систему крові (у тому числі містять ацетилсаліцилову кислоту);

Застосування препаратів, що впливають на метаболізм та виведення варфарину (антибіотики, протидіабетичні препарати та ін.). Якщо вам необхідний прийом нового лікарського препарату, обов'язково зверніться до лікаря, повідомивши йому, що ви приймаєте варфарин;

Занять травматичними видами спорту, де можливі удари, забиті місця, падіння;

Внутрішньом'язові уколи. При амбулаторному лікуванні в більшості випадків можна підібрати ліки для прийому внутрішньо;

Повторне приймання ліків протягом одного дня. Якщо не пам'ятаєте, чи приймали сьогодні варфарин, пропустіть прийом;

Зміни харчування.

Варфарин діє на згортання крові через вітамін К, який у різних кількостях міститься в їжі. Не треба уникати продуктів із високим вмістом вітаміну К! Харчування має бути повноцінним. Необхідно тільки стежити, щоб не було значної зміни їх частки в раціоні, наприклад, залежно від сезону. Якщо ви значно збільшите вживання харчових продуктів, багатих на вітамін К на тлі підібраної стабільної дози варфарину, це може сильно послабити його дію і призвести до тромбоемболічних ускладнень. Максимальна кількість вітаміну К (3000-6000 мкг/кг) міститься у темно-зелених листових овочах та травах (шпинаті, петрушці, зеленій капусті), а у зеленому чаї до 7000 мкг/кг; проміжна кількість (1000-2000 мкг/кг) - у рослинах з блідим листям (білокачанної капусті, салаті, броколі, брюссельській капусті). Значна кількість вітаміну міститься в бобових, майонезі (за рахунок рослинних олій), зеленому чаї.

Закінчення табл. 3.7

Запитання, на які необхідно дати відповідь хворому

Конкретна інформація для хворого щодо певного лікарського засобу

Жири та олії містять різну кількість вітаміну К (300-1000 мкг/кг), більше його в соєвій, рапсовій, оливковій олії. Вміст вітаміну К у молочних, м'ясних, хлібобулочних виробах, грибах, овочах та фруктах, чорному чаї, каві низька (не більше 100 мкг/кг). Регулярне вживання ягід та соку журавлини може посилити дію варфарину.

Невеликі дози алкоголю при нормальній функції печінки не впливають на терапію антикоагулянтами, проте до вживання спиртного слід ставитись обережно.

Прийом полівітамінів, що містять вітамін К, може послабити дію варфарину

Що робити, якщо пропущено дозу лікарського засобу

Прийняти препарат наступного дня

Де отримати більш детальну інформацію про лікарський засіб

Інструкція, ТКФЗ Державного реєстру ЛЗ

Виконайте завдання для самостійної роботи

Завдання 3.9. Відповідно до викладеного в табл. 3.7 планом сформулюйте рекомендації для хворого на застосування будь-якого ЛП. Використовуйте інструкцію з медичного застосування ЛП та ТКФЗ.

У США інструкції ЛП для пацієнтів розроблені Національним інститутом здоров'я та знаходяться у вільному доступі в Інтернеті: http://www.nlm.nih.gov/medlineplus/druginformation.ritml

Чи існують додаткові методи покращення комплаентності?

Ефективними методами покращення комплаентності також вважаються:

Ведення щоденників самоспостереження;

- Розробка для пацієнта індивідуального плану лікування;

Відвідування шкіл пацієнтів із різними захворюваннями. Дуже важливо, щоб пацієнт не лише акуратно здійснював

прийом ЛЗ, але й міг адекватно самостійно оцінювати свій стан, вживати необхідних заходів та вчасно звертатися до лікаря. Для цього корисне ведення щоденника самоспостереження (табл. 3.8), де пацієнт фіксував би свої скарги, ключові лабораторно-інструментальні показники (рівень артеріального тиску при артеріальній гіпертензії, обсяг діурезу при серцевій недостатності, температуру тіла при інфекційному захворюванні, рівень глюкози при цукровому діабеті і ін.). , а також додатковий прийом ЛЗ. Подібний щоденник дозволить лікарю більш адекватно оцінити динаміку стану та ефективність терапії, що проводиться, а також підвищить прихильність пацієнта до лікування.

Таблиця 3.8.Щоденник самоспостереження пацієнта із бронхіальною астмою

Виконайте завдання для самостійної роботи

Завдання 3.10. Використовуючи як приклад табл. 3.8, розробте щоденник самоспостереження для пацієнта за профілем вашої майбутньої спеціальності.

Ще одним важливим компонентом поліпшення комплаентності вважається складання індивідуального плану лікування, де пацієнту буде надано рекомендації про те, у яких випадках необхідно самостійно змінити прийом препаратів, коли слід негайно звернутися до лікаря і що робити в екстрених ситуаціях (табл. 3.9).

Таблиця 3.9.Індивідуальний план лікування хворого на бронхіальну астму

I. Ваше основне лікування:

Щодня приймайте:

1. 25 мкг салметеролу та 125 мкг флутиказону по 2 вдихи вранці та ввечері.

2. Зафірлукаст по 10 мг вранці та ввечері внутрішньо, за 1 год до їди або 2 год після їди.

3. Перед фізичним навантаженням прийміть: сальбутамол 0,2 мг 1-2 вдихи за 15-30 хв. У разі виникнення симптомів «на вимогу» прийміть: сальбутамол 0,2 мг 1-2 вдихи.

ІІ. Коли потрібно збільшити лікування?

Дайте відповідь на запитання, проаналізувавши свій стан за останній тиждень:

З'являлися симптоми (задишка, кашель, свисти, сором у грудях і

пр.) більше 2 разів на день?

Через астму ви прокидалися вночі?

Вам доводилося використовувати інгалятор на вимогу більше 2 разів? Ваша фізична активність знизилася через астму?

Рівень ПСВ став нижче л/хв?

Якщо ви відповіли "ТАК" 3 рази або більше, вам необхідно збільшити лікування.

Щодня приймайте:

1. 25 мкг салметеролу та 250 мкг флутиказону по 2 вдихи вранці та ввечері.

2. Зафірлукаст по 20 мг вранці та ввечері внутрішньо, за 1 год до їди або 2 год після їди.

3. Оцінюйте свій стан щодня, дотримуйтесь цієї схеми лікування протягом тижня.

ІІІ. Коли йти на прийом до лікаря?

Якщо протягом тижня покращення не настає, запишіться на прийом:

_(вказати телефон)_(реєстратура)

Лікар: П.І.Б._

IV. Екстренна ситуація.

У вас з'явилася важка задишка, ви можете вимовити лише короткі речення.

У вас тяжкий напад астми і ви налякані.

Ви використовуєте інгалятор «на вимогу» кожні 4 години, а поліпшення не настає.

Зробіть 2-4 вдихи сальбутамолу. Прийміть 20 мг метилпреднізолону внутрішньо. Зверніться за допомогою «03».

Продовжуйте інгаляції сальбутамолу по 2-4 вдихи кожні 20-30 хв до приїзду лікаря.

Виконайте завдання для самостійної роботи

Завдання 3.11. Використовуючи наведений нижче приклад індивідуального плану лікування хворого на бронхіальну астму (див. табл. 3.9 з Global Initiative for Asthma, GINA, 2006), розробіть індивідуальний план лікування пацієнта за вашим майбутнім фахом.

Ще один спосіб підвищення комплаентності та мотивації хворих до лікування - це створення спеціальних шкіл здоров'я для хворих на певні захворювання (школа здоров'я для хворих на цукровий діабет, артеріальну гіпертензію, бронхіальну астму, хронічний вірусний гепатит, виразкову хворобу, остеопороз, міопію та ін.). Основні завдання шкіл:

1. Дати пацієнтові уявлення про його захворювання, розповісти про сучасні можливості лікування.

2. Дати можливість пацієнту оцінити тяжкість свого стану та адекватність лікування, що проводиться.

3. Навчити пацієнта самостійно розпізнавати погіршення, що наближається, і запобігати його.

4. Навчити пацієнта самостійної допомоги при загостренні, що розвинулося.

5. Пояснити призначення приладів індивідуального користування (наприклад, спейсера, пікфлоуметра, глюкометра, тонометра та ін.).

6. Навчити ведення щоденника здоров'я.

8. Сформувати у пацієнта навички «відповідального самолікування». Під самолікуванням (за визначенням експертів ВООЗ) розуміють «розумне застосування самим пацієнтом ЛЗ, які перебувають у вільному продажу, з метою профілактики чи лікування легких розладів здоров'я до надання професійної лікарської допомоги». Слід інформувати пацієнта, що «самолікування може бути використане лише при обмеженій кількості нездужань» і має бути обмеженим у часі. У числі ЛЗ, які можуть використовуватися при самолікуванні, повинні бути лише безрецептурні ЛЗ, що застосовуються хворим суворо за інструкцією із застосування ЛП, тобто. листку-вкладишу: показання, протипоказання, дози, шляхи введення, кратність застосування, тривалість застосування та ін.

Приклад плану занять у школі хворих на цукровий діабет другого типу, що проводилися на базі клініки ендокринології Московської медичної академії ім. І.М. Сєченова (табл. 3.10).

Таблиця 3.10.План занять у школі хворих на цукровий діабет другого типу Заняття 1. Тема «Що таке ЦД»

Основні поняття діабетології (нормальний, високий та низький рівні цукру в крові, нирковий поріг коми). Цілі лікування.

Симптоми ЦД та причина їх появи.

«Чотири кити» комплексної терапії ЦД другого типу. Заняття 2. Тема «Самоконтроль»

Що входить у поняття самоконтролю і чому він необхідний.

Способи самоконтролю цукру у крові, сечі, ацетону в сечі.

Навчання практичним навичкам проведення самоконтролю.

Ведення «Щоденника хворого на ЦД другого типу». Заняття 3, 4. Тема «Основні принципи дієтотерапії при ЦД другого типу»

Поняття про енергетичний баланс здорової людини та пацієнта з

ЦД другого типу.

Основні складові їжі та поняття про калорійність. Методи зменшення калорійності добового раціону. Вуглеводи як основне джерело енергії, класифікація вуглеводів. Система «хлібних одиниць», взаємозамінність продуктів, які містять вуглеводи. Харчовий "світлофор".

Вживання цукрозамінників та підсолоджувачів. Як бути із алкоголем? 5. Тема «Фізичні навантаження. Гіпоглікемія»

Частина I. Фізичні навантаження:

Необхідність розширення фізичної активності при ЦД другого типу;

Основні правила дозування фізичних навантажень;

Поведінка перед початком та у процесі фізичного навантаження. Частина ІІ. Гіпоглікемія:

Що таке гіпоглікемія;

Причини розвитку гіпоглікемії;

Симптоми легкої та тяжкої гіпоглікемії;

Що робити у разі виникнення симптомів гіпоглікемії.

Закінчення табл. 310

Виконайте завдання для самостійної роботи

Завдання 3.12. Використовуючи наведений приклад, розробіть план занять у школі пацієнта з вашої майбутньої спеціальності.

Заняття 6. Тема «Пізні ускладнення ЦД: ретинопатія, нефропатія, нейропатія»

Обговорення ускладнень ЦД, що найчастіше зустрічаються, і причин їх виникнення, введення термінів «ретинопатія», «нефропатія», «нейропатія».

Ретинопатія: стадії, симптоми, профілактика, лікування. Нефропатія: стадії, симптоми, профілактика, лікування. Нейропатія: форми, симптоми, профілактика, лікування. Заняття 7. Тема «Правила догляду за ногами»Причини ураження ніг при ЦД.

Профілактика ураження ніг (правила «можна-не можна»).

Навчання навичкам огляду взуття та поведінки при виникненні

ушкоджень ніг. Заняття 8. Тема «Артеріальна гіпертензія, атеросклероз та ЦД другого типу»

Частина I. Артеріальна гіпертензія:

Що таке артеріальний тиск;

Нормальний рівень АТ, причини та механізми підвищення АТ;

Симптоми підвищеного артеріального тиску;

Небезпеки підвищеного артеріального тиску;

Принципи сучасної антигіпертензивної терапії. Частина ІІ. Атеросклероз:

Що таке атеросклероз та механізми його розвитку;

Нормальний рівень холестерину у крові;

Прояви атеросклерозу;

Принципи сучасної гіполіпідемічної терапії. Заняття 9. Тема «Інсулінотерапія при ЦД другого типу»

Причини призначення інсуліну при ЦД другого типу. Механізм дії інсуліну.

Види препаратів інсуліну та системи для введення інсуліну. Схеми інсулінотерапії. Навчання техніки ін'єкцій.

Виконайте завдання для самостійної роботи

Завдання 3.13. Проаналізуйте випадок із клінічної практики. Пацієнт К. 58 років спостерігається у кардіолога за місцем проживання з діагнозом: ІХС, стенокардія напруги II ФК-класу. Постійна форма миготливої ​​аритмії. Артеріальна гіпертензія 3-го ступеня, дуже високий ризик. Дифузно-вузловий зоб, еутиреоз. Виразкова хвороба шлунка, ремісія. У зв'язку з наявністю у хворого на постійну форму миготливої ​​аритмії для профілактики тромбоемболічних ускладнень кардіологом призначено варфарин, під контролем МНО підібрано дозу варфарину 7,5 мг на добу, на фоні прийому якої МНО становило 2,37-2,5. Біль у животі, геморагій не відзначав. Через 3 міс після початку прийому варфарину хворий доставлений бригадою швидкої допомоги з громадського місця в стаціонар у зв'язку з болями, що розвинулися, в прекардіальній ділянці (загальна тривалість нападу 10 хв), купованими після 4-кратного застосування спрею нітрогліцерину (за словами хворого). При надходженні даних за гострий коронарний синдром був, клінічна картина геморагічного синдрому була відсутня. Хворий оглянутий лікарями терапевтами приймального відділення та черговим терапевтом. Призначено обстеження (клінічний аналіз крові, загальний аналіз сечі, реакція Вассермана, ЕКГ, біохімічний аналіз крові, МНО), лікування (ізосорбіду динітрат, варфарин – 7,5 мг на добу, метопролол, еналаприл, індапамід). Однак хворий цього ж дня самостійно залишив відділення після огляду чергового терапевта, у зв'язку з чим призначене обстеження та лікування розпочато не було. Через місяць хворого доставлено бригадою швидкої медичної допомоги до стаціонару з явищами геморагічного синдрому: підшкірні крововиливи, субкон'юнктивальний крововилив, гематурія, МНО при цьому становило 9,8. За словами хворого, протягом останнього місяця приймав варфарин у колишній дозі (7,5 мг на добу), проте МНО не контролював (пропустив плановий вимір МНО), оскільки перебував за містом, до лікарів не звертався. Також хворий стверджує, що протягом останнього місяця зловживав алкоголем.

У стаціонарі варфарин був скасований на тлі переливання свіжозамороженої плазми, застосування препарату вітаміну К. Геморагічний синдром був купований, МНО знизилося і при виписці становило 1,12. Хворий виписаний у задовільному стані з рекомендаціями не приймати варфарин через неможливість контролювати МНО.

1. Які дії хворого сприяли розвитку геморагічного синдрому?

2. Які причини низького комплаєнсу у хворого?

3. Як можна уникнути розвитку геморагічного синдрому у хворого?

Клінічна фармакологія Загальні питання клінічної фармакології. Практикум: навчальний посібник. Сичов Д.А., Долженкова Л.С., Прозорова В.К. та ін / За ред. В.Г. Кукес. 2013. – 224 с.: іл.

Розподіл, біотрансформацію та виведення лікарських засобів?

Фармакодинаміка.

Фармакокінетика.

2.Основний механізм всмоктування лікарських засобів у шлунково-кишковому тракті:

Активний транспорт

Полегшена дифузія.

Пасивна дифузія через мембрани клітин.

Піноцитоз.

3.Основне місце всмоктування ліків – слабких підстав:

Шлунок.

Тонка кишка.

4.Основне місце всмоктування ліків – слабких кислот:

Шлунок.

Тонка кишка.

5. Який спосіб введення лікарських засобів забезпечує 100% біодоступність?

Внутрішньом'язовий.

Ректальний.

Внутрішньовенний.

Через рота.

6.Як зміниться всмоктування лікарських засобів – слабких кислот при зменшенні кислотності шлункового соку?

Збільшиться.

Зменшиться.

7.Як зміниться всмоктування лікарських засобів – слабких підстав при зменшенні кислотності шлункового соку?

Збільшиться.

Зменшиться.

8.Шляхом пасивної дифузії через біологічні мембрани легко транспортуються речовини:

Ліпофільні.

Полярні.

Гідрофільні.

9. Ентеральний шлях введення лікарських засобів:

Внутрішньом'язовий.

Інгаляційний.

Сублінгвальний.

Внутрішньовенний.

10. Парентеральний шлях введення лікарських засобів:

Через рота.

У пряму кишку.

Підшкірний.

Сублінгвальний.

11.Де в основному відбуваються всмоктування більшої частини лікарських засобів?

У ротовій порожнині.

У шлунку.

У тонкому кишечнику.

У товстому кишечнику.

12.Внутрішньовенно можна вводити:

Олійні розчини.

Нерозчинні сполуки.

Осмотично активні сполуки.

Мікрокристалічні суспензії.

Нерозчинні сполуки.

13. Як називається розділ фармакології, що вивчає види дії лікарських засобів, фармакологічні ефекти, механізм дії?

Фармакодинаміка.

Фармакокінетика.

14. Яку функціональну зміну в організмі викликають серцеві глікозиди при серцевій недостатності?

Порушення.

Пригнічення.

Тонізація.

Заспокоєння.

15. Яка функціональна зміна в організмі викликає засіб, що знижує артеріальний тиск при артеріальній гіпертензії?

Порушення.

Пригнічення.

Тонізація.

Заспокоєння.

16.Як називається накопичення в організмі лікарського засобу за його повторних введень?

Функціональна кумуляція.

Сенсибілізація.

Матеріальна кумуляція.

Тахіфілаксія.

17. Толерантність – це:

Алергічна реакція організму на повторне введення ліків.

Зменшення фармакологічного ефекту повторне введення ліків.

Непереборне прагнення повторного прийому ліків.

18. Зниження ефекту при введенні лікарських засобів через короткі проміжки часу – це:

Тахіфілаксія.

Ідіосинкразія.

Сенсибілізація.

Пристрасть.

19. Побічний ефект, який може виникнутитільки при повторних введеннях лікарських засобів:

Ідіосинкразія.

Тератогенна дія.

Мутагенна дія.

Звикання.

20. Побічний ефект, який може виникнутитільки при застосуванні психотропних засобів:

Ідіосинкразія.

Пристрасть.

Звикання.

Сенсибілізація.

21.Визначте вид взаємодії лікарських засобів: хворому під час отруєння мускарином проведено промивання шлунка суспензією активованого вугілля:

Підсумований синергізм.

Хімічний антагонізм.

Конкурентний антагонізм.

Фізичний антагонізм.

22.Мутагенна дія – це:

23.Тератогенна дія – це:

Пошкодження генетичного апарату зародкової клітки.

Порушення диференціювання тканин плода, що викликає різні аномалії.

Побічний ефект, що виникає в перші 12 тижнів після запліднення та викликає загибель зародка.

24. Ембріотоксична дія – це:

Пошкодження генетичного апарату зародкової клітки.

Порушення диференціювання тканин плода, що викликає різні аномалії.

Побічний ефект, що виникає в перші 12 тижнів після запліднення та викликає загибель зародка.

25. Взаємне посилення ефекту одного лікарського засобу іншим називається:

Синергізм.

Антагонізм.

26. Взаємне послаблення ефекту одного лікарського засобу іншим називається:

Синергізм.

Антагонізм.

27.Яким терміном позначається дія лікарських засобів під час вагітності, що призводить до вродженої потворності?

Мутагенний.

Ембріотоксичний.

Тератогенний.

28. Призначення лікарських засобів з метою усунення причин захворювання називається:

Патогенетична терапія.

Етіотропна терапія.

Симптоматична терапія.

29.Як називається взаємодія двох лікарських засобів, що відбувається на рівні рецепторів одного типу і що призводить до послаблення ефекту?

Потенційований синергізм.

Підсумований синергізм.

Конкурентний антагонізм.

30. Інгаляційний газоподібний наркозний засіб.

Фторотан.

Енфлуран.

Гексенал

Оксид азоту.

31. Інгаляційний наркозний засіб, що широко використовується при наданні першої допомоги хворим з великими травмами, інфарктом міокарда, при пологах.

Ефір для наркозу.

Фторотан.

Тіопентал-натрій.

Оксид азоту.

32. Наркозний засіб при короткочасних втручаннях, що не потребують вираженого м'язового розслаблення, при обробці опікових ран, перев'язках.

Кетамін.

Гексенал

Пропанидид.

Натрію оксибутират.

33.Снодійний засіб, похідний бензодіазипіну.

Фенобарбітал.

Нітразепам.

Натрію оксибутират.
34.Снодійний засіб, похідний барбітурової кислоти.

Флунітразепам.

Фенобарбітал.

35.Снодійний засіб, що не залишає після сну млявості, сонливості, порушення працездатності.

Фенобарбітал.

Нітразепам.

Мідазолам.

36. Побічний ефект, що обмежує прийом барбітуратів та похідних бензодіазепінів як снодійні засоби.

Млявість, сонливість, апатія.

Лікарська залежність (психічна, фізична).

Алергічні реакції.

37. Засіб, що застосовується для усунення судом.

Сибазон.

Аміназин.

Фенобарбітал.

38.До якої фармакологічної групи відносять морфін, промедол, омнопон, фентаніл?

Ненаркотичні аналгетики.

Транквілізатори.

Психостимулятори.

Наркотичні аналгетики.

39. Вкажіть рецептори, куди діють наркотичні анальгетики.

Адренорецептори.

Холінорецептори.

Опіоїдні рецептори.

40. Для яких аналгетиків характерні протитривожний та ейфоризуючий ефекти?

Ненаркотичні аналгетики.

Наркотичні аналгетики.

41.Как впливають наркотичні анальгетики на тонус гладком'язових органів?

Надають спазмолітичну дію.

Надають спазмогенну дію.

Чи не впливають на тонус гладком'язових органів.

42. Вплив наркотичних аналгетиків на кашльовий центр.

Пригнічують кашльовий центр.

Чи не впливають на кашльовий центр.

43.Наркотичний аналгетик, тривалість дії якого 30 хвилин.

Промедол.

Фентаніл.

Пентазоцин.

44. Показання до застосування наркотичних аналгетиків.

Головний біль.

Зубний біль.

М'язові болі.

Тяжкі травми, опіки та поранення.

45.Морфін або фентаніл при інфаркті міокарда переважно вводити

46.При болях спастичного характеру (ниркова колька та жовчнокам'яна хвороба) наркотичні анальгетики обов'язково поєднують

з ненаркотичними анальгетиками

З холіноблокаторами або міотропними спазмолітиками.

47.М-холіноблокатор.

Платіфілін.

Норадреналіну.

48.Спазмолотик міотропної дії при болях спастичного характеру.

Но-шпа (дротаверину гідрохлорид).

Пентамін.

Празозин.

49. Визначте групу за побічними ефектами: психічна та фізична залежність, пригнічення дихального центру, обстипація (запор), бронхоспазм, брадикардія:

Нейролептики

Ненаркотичні анальгетики

Наркотичні анальгетики

Транквілізатори

50. Ненаркотичний аналгетик – похідне саліцилової кислоти.

Парацетамол.

Анальгін.

Ацетилсаліцилова кислота.

Диклофенак (ортофен)

51.Які лікарські засоби мають наступні ефекти: болезаспокійливий, жарознижувальний, протизапальний?

Наркотичні аналгетики.

Транквілізатори.

Седативні засоби.

Ненаркотичні аналгетики.

Гальмування синтезу простагландинів.

Порушення опіоїдних рецепторів ЦНС.

53. Ненаркотичний аналгетик з найбільш вираженим знеболюючим ефектом.

Кеторолак.

Індометацин.

Анальгін.

Парацетамол.

54.Нестероїдні протизапальні засоби при лікуванні запалення суглобів, м'язів, нервових стволів, а також ревматизму.

Індометацин, диклофенак.

Промедол, пентазоцин.

Преднізолон, дексаметазон.

55. Ненаркотичний аналгетик, що не має протизапальної дії.

Анальгін.

Парацетамол.

Індометацин.

56. Найбільш ефективний ненаркотичний анальгетик, що застосовується при забитих місцях кісток і суглобів, розтягування зв'язок, вивихах і т.п.

Анальгін.

Ібупрофен.

Кеторолак.

57. Комбінований препарат, що застосовується при спазмі сечовивідних, жовчовивідних шляхів (коліки).

Баралгін.

Цитрамон.

Пенталгін.

58. Побічний ефект ненаркотичних аналгетиків, пов'язаний з пригніченням синтезу простагландинів.

Алергічні реакції.

Нудота блювота.

Виникнення виразок шлунка (ульцерогенна дія).

Запаморочення.

59. Ненаркотичний аналгетик, що застосовується як антиагрегант для попередження тромбоутворення при ішемічній хворобі серця.

Анальгін.

Індометацин.

Ацетилсаліцилова кислота.

60. Побічний ефект, найбільш характерний для анальгіну.

Шум і дзвін у вухах.

Кровотеча з ясен внаслідок порушення зсідання крові.

Порушення кровотворення (лейкопенія, агранулоцетоз, тромбоцитопенія).

Алергічні реакції.

61.Аміназін - це:

Психостимулятор.

Антидепресант.

Нейролептик.

Транквілізатор.

62. Який психотропний ефект викликають нейролептики?

Антипсихотичний.

Анксіолітичний.

Антидепресивний.

63. Антипсихотичний ефект характеризується:

Усунення психомоторного збудження.

Поліпшенням розумової та фізичної працездатності.

Усунення марення та галюцинацій.

64.Протиблювотний ефект має:

Етаперазин.

Фенезепам.

Амітриптілін.

Сіднокарб.

65.Феназепам, сибазон, хлозепід, тофізопи - це:

Нейролептики.

Транквілізатори.

Ноотропи.

Седативні засоби.

66. Яка група психотропних засобів вибірково усуває тривогу, страх, явища емоційної нестійкості?

Антидепресанти.

Психостимулятори.

Нейролептики.

Транквілізатори.

67. Механізм дії транквілізаторів пов'язаний:

З блокадою дофамінових рецепторів у мозку.

Зі збудженням адренорецепторів у мозку.

З підвищенням чутливості ГАМК-рецепторів до гальмівного медіатора мозку ГАМК (гаммааміномасляна кислота).

68. Основний ефект транквілізаторів:

Анксіолітичний (протитривожний).

Психоседативний.

Антипсихотичний.

69.Транквілізатор, що не має седативного ефекту (денний):

Феназепам.

Альпразол.

Тофізопи.

70.Седативний ефект транквілізаторів наводить:

До зниження швидкості та точності реакції, сонливості, падіння розумової працездатності.

До підвищення швидкості та точності реакції, сонливості, падіння розумової працездатності.

71.Вкажіть непсихотропний ефект транквілізаторів.

Анксіолітичний.

Протисудомний.

Психоседативний.

72.Транквілізатори застосовують:

Неврози, невротичні та панічні реакції.

Депресії.

73.При стресових ситуаціях у здорових людей краще використовувати транквілізатори:

З седативним та міорелаксуючим ефектом (феназепам).

Без вираженого седативного та міорелаксуючого ефекту (тофізопи).

74. Побічним ефектом, що обмежує широке застосування транквілізаторів, є:

Психічна та фізична залежність.

Звикання.

Сонливість.

М'язова слабкість.

75.Кошти, які надають заспокійливу дію рахунок зниження возбудимости ЦНС:

Нейролептики.

Транквілізатори.

Седативні засоби.

Психостимулятори.

76.Препарати валеріани, собачої кропиви, пасифлори, півонії, броміди - це:

Психостимулятори.

Транквілізатори.

Ноотропи.

Седативні засоби.

77. Комбінований седативний препарат:

Корвалол.

Цитрамон.

Екстракт валеріани.

78.Седативні засоби застосовують:

Для лікування психозів.

Для лікування депресії.

При легких невротичних станах.

79. До антидепресантів відносять:

Аміназин.

Амітриптілін.

Феназепам.

Сіднокарб.

80. Основний психотропний ефект антидепресантів:

Тимолептичний (покращення патологічно зміненого настрою).

Седативний.

Психостимулюючий.

81. Антидепресанти застосовують:

Для лікування психозів.

Для лікування неврозів.

Для лікування депресії.

82.Сіднокарб, кофеїн, бемитил - це:

Психостимулятори.

Нейролептики.

Седативні засоби.

83. Основний ефект психостимуляторів:

Анксіолітичний.

Психоседативний.

Антидепресивний.

Психостимулюючий.

84.Психостимулюючий ефект проявляється:

Підвищенням фізичної та розумової працездатності.

Зниженням фізичної та розумової працездатності.

85.За механізмом дії сіднокарб є:

Адреноміметиком непрямої дії.

Адреноміметиком прямої дії.

Адреноблокатор прямої дії.

86. Ноотропний засіб:

Пірацетам.

Феназепам.

Аміназин.

87.Кошти, що покращують процеси пам'яті, навчання:

седативні.

Транквілізатори.

Ноотропи.

88.Препарати з лимонника китайського, левзеї, жень-шеня, елеутерококу, родіоли є:

Загальтонізуючими засобами.

седативними засобами.

89.М'який психостимулюючий ефект родіоли проявляється:

У підвищенні розумової та фізичної працездатності, в ослабленні втоми.

У зниженні розумової та фізичної працездатності.

90.Ефект загальнотонізуючих засобів проявляється:

Після одноразового застосування.

Після застосування протягом 4-6 тижнів.

Вітаю, дорогі друзі!

Ось ще одна тема у нас з вами пройдена. Дуже сподіваюся, що я допомогла вам розібратися з лікарськими препаратами.

Хочете перевірити себе, чи всі зрозуміли? Чи все вклалося «по поличках»?

У такому разі нижче буде тест, який я вам приготувала.

Як завжди, постаралася зробити його максимально практичним.

У деяких питаннях фігурують препарати, які ми з вами не розбирали докладно. Це я зробила спеціально. А хто сказав, що знати їх не треба? 🙂

Якщо важко, буде привід заглянути в інструкцію.

Не засмучуйтесь, якщо результат виявиться не дуже...

Мені чомусь здається, що тут, на моєму блозі, бувають тільки ті трудяги фармбізнесу, які хочуть розібратися в усьому досконало. Про це говорять ваші запитання у коментарях. Тож не опускайте рук і вивчайте все, що я тут викладаю. Якщо потрібно, проходьте ту саму тему ще й ще раз.

Я вже казала вам, що я люблю тести? Це класна форма перевірки знань, щоб зрозуміти, над чим потрібно попрацювати.

Мені за своєю основною роботою доводиться їх складати часто.

Не можу сказати, що співробітники компанії, в якій працюю, від них у захваті. Вони завжди напружуються, оскільки від результатів тестування що-небудь залежить.

Але тут, на блозі, ніяких нервів не буває. Його можна проходити хоч 100 разів, щоб запам'ятати та зрозуміти, що до чого. Тим більше, обмежень за часом ніяких.

А отже, є достатньо часу, щоб подумати над питанням і поговорити без поспіху.

Наприклад, ось таке питання:

«Відвідувач скаржиться на сильну свербіж, печіння в області ануса, іноді буває кров у стільці. Який препарат Ви запропонуєте насамперед?»

Ход думок буде приблизно таким:

«Якщо сильна сверблячка і печіння, значить, потрібно щось із гормональним компонентом. Тому препарати X та Y відкидаємо.

Залишається два. Якщо буває кров у стільці, значить, препарат із гепарином краще не давати. Відкидаємо і препарат Z. Відтак залишається препарат...»

Загалом, спокійненько проходьте тест, бачите неважнецький результат, отже треба щось ще раз почитати, а потім пройти знову і знову, поки результат вас задовольнить.

Отже, у тесті 18 питань. У кожному можливий лише ОДИН правильний варіант відповіді. Максимальна кількість балів, які можна набрати – 18.

Бажаю удачі!

Як Вам сподобався цей тест? А свої результати? 🙂

Розкажіть, що вийшло! Було складно?

Повну збірку тестів у форматі «word» з відповідями ви можете придбати.

А я з вами прощаюсь до нової зустрічі на блозі «Новини».

З любов'ю до вас, Марино Кузнєцова

Виберіть правильне твердження: а) біодоступність-кількість ЛЗ, що надходить у системний кровотік, виражена у відсотках від введеної дози, б) біодоступність визначається величиною адсорбції ЛЗ у ШКТ і вираженістю ефекту першого проходження через печінку. (в/м або всередину)/AUC (в/в). г) біодоступність ЛЗ при внутрішньом'язовому введенні визначається ступенем його всмоктування та біотрансформації в організмі.
Відповідь: а Б В

2.
Відповідь: Атровент

3.

Відповідь: а, д

4.

Відповідь:

5.
Відповідь:

6.

д) Ксіліт
Відповідь: а,в

7.

Відповідь: а, б, д

8.
Відповідь:

9. надійшов з гострим інфарктом міокарда, що виник 5год тому. Призначення: анаприлін 20 мг 4 рази на добу всередину, гепарин внутрішньовенно крапельно по 10 000 ОД кожні 4 години. При цьому вдалося досягти збільшення часу зсідання крові до 18-23 хвилини. На наступний день у б-ного діагностована правостороння нижньодолева пневмонія. Призначена натрієва сіль бензилпеніциліну (по 1000 000 ОД кожні 4 год) в/в. Через 4 години час згортання крові склало 8 хвилин. Яка ваша тактика?
Відповідь:

10.

11.
Відповідь: Віт.В12 у дозі 500 мкг/добу через день, фолієва кислота в дозі 1,5 мг/добу, сульфат заліза(80 мг Fe2+) 1 раз на добу

12.

Відповідь: Віт.С

13.

Відповідь: Церебролізин

14.
алергію (на бутадіон, гепарин, метиндол, пеніцилін, теофілін). У стаціонарі б-ної призначені реопірин по 5 мл в/м 1 раз на добу, гідрокортизону гемісукцинат по 100 мг в порожнину колінних суглобів, тавегіл по 0,001. Через 3 дні у б-ної з'явилися сверблячі еритематозні висипання на шкірі тулуба. Яка найбільш ймовірна причина
погіршення стану?
Відповідь:

15.



Відповідь: а, б, д, е, з, і

16.
Відповідь: Через кілька місяців

17.


Відповідь: а, б, в, д, е

18.
Відповідь: а, б, в, г, д, ж, з

19.
Відповідь:

20.
Відповідь: Ципрофлоксацин

21. Феномен першого проходження ЛЗ через печінку залежить від: а) кровопостачання печінки; б) зв'язування ЛЗ з білком; в) активності ферментів гепатоцитів; г) рівня екскреції ЛЗ; д) швидкості всмоктування
Відповідь: а, в

22. Препарати, що впливають на мікросомальні ферменти печінки: індуктори мікросомальних ферментів печінки: а) пеніцилін; б) нітрогліцерин; в) фенобарбітал; г) фуросемід; д) бутадіон; е) кортизол; ж) пропранолол; ) дифенін
Відповідь: в, д

23. надійшла у відділення з ж-ми на болі у правій молочній залозі, підвищення Т. до 39,5 С. ​​Захворіла 3 дні тому, на 10-й день після пологів. При надходженні у відділення у верхньозовнішньому квадранті правої молочної залози виявлено гіперемію шкіри, масивний інфільтрат з флуктуацією в центрі. Діагноз: гострий правобічний мастит. Взято посів рани, що відокремлюється. Визначте антибіотик першого вибору
Відповідь: Цефазолін

24.

Відповідь: Анафілактична реакція

25.

Відповідь: Левоміцетин

26.
Відповідь:

27.
Відповідь: Бігуаніди

28.

Відповідь: Гіпотонія, запаморочення.

29.

Оцініть дії лікаря.

30. У хворого Д., 53 років, діагноз: ІХС, стабільна стенокардія? ФК, постінфарктний кардіосклероз, миготлива аритмія, ХНК? Приймав строфантин, дигоксин, фуросемід, панангін у середніх терапевтичних дозах. Несподівано у хворого підвищилася температура до 38,4 ° С, з'явилася кашель, задишка, крепітація в легенях праворуч. На рентгенограмі легень праворуч у нижній частці визначається ділянка інфільтрації. До лікування додані гентаміцин, сульфокамфокаїн, супрастин.

Відповідь:

31.

Відповідь: Фентоламін.

32.

Відповідь: г, д

33.

Відповідь:

34.
Відповідь:

35.
Відповідь: Еналаприл.

36.

Відповідь: а, б, г

37.

Відповідь:

38.
Відповідь:

39.
Відповідь:

40.

Відповідь: все перераховане

41. Препарати, що впливають на мікросомальні ферменти печінки: інгібітори мікросомальних ферментів печінки:
а) пеніцилін; б) нітрогліцерин; в) фенобарбітал;
г) фуросемід, д) бутадіон, е) кортизол, ж)пропранолол,
з) циметидин; і) левоміцетин; к) дифенін
Відповідь: з,і

42.
Відповідь: Через 7-14 днів

43. Вкажіть комбінацію препаратів, що призводить до виникнення конкуренції за зв'язування з білком, що може зумовити небезпечне підвищення вмісту вільної фракції одного з препаратів у крові та появу симптомів його передозування.
Відповідь: неодикумарин та бутадіон

44. Виберіть ЛЗ з вузьким терапевтичним діапазоном:
а) пеніциліни; б) протисудомні засоби,
в) антиаритмічні засоби; г) дигоксин; д) метотрексат; е) теофілін; ж) циклоспорин; з) макроліди.
Відповідь: б,в,г,д,е,ж

45. Вкажіть комбінації препаратів, при яких внаслідок конкуренції за зв'язок з білком відбувається збільшення концентрації в плазмі крові вільної фракції одного з них: а.строфантин і місклерон, б.дигітоксин та місклерон, в.
Відповідь: б,в

46. виявлені часта шлуночкова екстрасистолія та пароксизми миготливої ​​аритмії. ЧСС 74 в хв, АТ 140/80 мм. рт. ст. Останні 3 г. турбують напади стенокардії напруги та спокою. -ного: а) Хінідін,
б) Боннекор, в) Етацизін,

Відповідь: а,б

47. Відомо, що при комбінації хінідину та дигоксину часто спостерігається глікозидна інтоксикація. З чим вона пов'язана? Фармакодинамічна взаємодія:
Відповідь: синергізм

48. Відомо, що при комбінації хінідину та дигоксину спостерігається глікозидна інтоксикація. З чим вона пов'язана? Фармакокінетична взаємодія, вплив хінідину на:
Відповідь: зв'язок з білком

49. Критичні періоди внутрішньоутробного розвитку:
а. період передімплантаційного розвитку (1 тиж)
б. стадія ембріогенезу закінчується до 8 тижнів.
в. стадія ембріогенезу закінчується до 8 місяців.
м. період безпосередньо перед пологами
Відповідь: а, б, г

50. Виберіть із нижченаведених препаратів, що мають перераховані властивості: Протимікробні препарати, застосування яких практично безпечне при вагітності: а.сульфаніламди, у тому числі бісептол,
б.аміноглікозиди,тетрацикліни,рифампіцини,метронідазол (в 1триместр вагітності), ст. пеніциліни, цефалоспорини, еритроміцин, лінкоміцин, фузидин, антимікотичні засоби, протипухлинні
антибіотики.
Відповідь: в

51. Метронідазол призначений матері-годувальниці, вкажіть побічні ефекти:
а.підвищення збудливості, тахікардія, б.пригнічення апетиту,блювота, в.пригнічення ЦНС,дихання,зниження маси тіла, р.збільшення секреції пролактину,нагрубання молочних залоз,д.гіпоплазія надниркових залоз,порушення обміну речовин,підвищення ризику розвитку білірубінової е. геморагії, порушення дихання, ацидоз, ж. пригнічення кровотворення, анемія, гіпотрофія, дисбактеріоз.
Відповідь: б

52. Антимікробні препарати першого вибору у новонароджених: а. бензилпеніцилін, оксацилін, карбеніцилін, гентаміцин, амікацин, б. (при неефективності цефалоспоринів першого покоління), г.еритроміцин, лінкоміцин, ністатин, леворин, карбеніцилін,
гентаміцин, сізоміцин
Відповідь: б

53.
Відповідь:

54. Основні особливості фармакокінетики ЛЗ в осіб похилого віку:
а.зниження швидкості абсорбції, б.прискорення абсорбції, в.зниження швидкості розподілу, г.прискорення розподілу, д.зменшення зв'язування ЛЗ з білками плазми, е.збільшення зв'язування ЛЗ з білками плазми, ж.уповільнення метаболізму, з.прискорення метаболізму,
і. уповільнення виведення ЛЗ, к. прискорення виведення ЛЗ.
Відповідь: а,в,д,ж,і

55.
Відповідь: б,в,г

56. Вкажіть побічні ефекти бета-адреноблокаторів: а) брадикардія, б) артеріальна гіпотензія, в) бронхоспазм, г) тахікардія, д) порушення функцій щитовидної залози,
е) переміжна кульгавість, ж) АВ-блокада
Відповідь: а, б, в, е, ж

57.
стану: а)Природний перебіг захворювання; б)Розвиток толерантності до нітратів; в)Синдром міжкоронарного обкрадання; г)Виникнення синдрому рикошету;
Відповідь: а,б

58. Вкажіть побічні ефекти аміодарону: а) брадикардія; б) артеріальна гіпотензія; в) бронхоспазм; г) тахікардія; д) порушення функцій щитовидної залози;
Відповідь: а,в,д,ж

59. Як зміниться ваша антиангінальна терапія, якщо у б-ного на тлі терапії нітратами виникне інсульт головного мозку?
Відповідь: відміна нітратів та призначення антиангінального препарату іншої групи

60. Які гіпотензивні препарати вважають найбільш безпечними для літніх хворих: а) бета-адреноблокатори, б) гангліоблокатори, в) симпатолітики, г) блокатори повільних кальцієвих каналів, д) тіазидні
діуретики; е) інгібітори АПФ.
Відповідь: г, д

61. Схема лікування кордароном:
Відповідь: за схемою, що передбачає поступове зниження дози з 600 мг до 200 мг на добу

62. Як впливають інгібітори МАО (антидепресанти) на пресорний ефект адреностимуляторів прямої та непрямої дії?
Відповідь: посилюють дію

63. Небензодіазепіновий» агоніст бензодіазепінових рецепторів:
Відповідь: Золпідем

64. Снодійний засіб - з'єднання аліфатичного ряду:
Відповідь: Хлоралгідрат

65.

Відповідь: А (б)

66. Протаміну сульфат призначають при передозуванні:
Відповідь: Гепарину

67. Який спосіб детоксикації найбільш ефективний при отруєнні речовинами, що пов'язують із білками та ліпідами крові?
Відповідь: Гемосорбція

68. Принцип дії налоксону при гострому отруєнні морфіном:
Відповідь: Перешкоджає дії морфіну на опіоїдні рецептори

69. Вкажіть препарати, що мають властивості антиоксидантів: а) верапаміл б) віт.А, в) віт.К, г) віт.С, д) віт.Е, е)селен, ж) карнозин, з) доксициклін
Відповідь: б, г, д, е, ж

70. Які ефекти характерні для нейролептиків?
а) антипсихотичний; б) седативний; в) протиблювотний.
Відповідь: а Б В

71. У Б-ої 64л. розвинувся гострий напад закритокутової глаукоми з сильними болями в правому оці з іррадіацією в голову. З'явилися нудота та блювання, задишка, виявлено ознаки гіпертонічного кризу 2 типу з ЧСС 62
в хв. АТ 200/140 мм.рт.ст.Б-ная протягом багатьох років страждає на гіпертонічну хворобу. У легенях велика кількість вологих дрібнопухирчастих хрипів. а. Клопамід, б.Верошпірон,в.Гіпотіазид, м.Фуросемід в/в, д. Діакарб:
Відповідь: г, д

72. протягом 15 років страждає на цукровий діабет, з приводу якого отримує інсулін по 70 ОД/добу, що підтримує рівень глікемії в межах 7,5-8,6 ммоль/л. Останнім часом стало підвищуватися АТ до
170/90-180/100 мм.рт.ст.в зв'язку з чим лікар призначив обзидан в добовій дозі 120 мг. Які побічні ефекти слід очікувати при даній комбінації ЛЗ? а.Гіперглікемія аж до коми, б.Серцева недостатність, в.Гіпоглікемія аж до коми, г.Ортостатична гіпотензія, д. Гіпертензія
Відповідь: б,в

73. , що страждає на гормонально-залежну бронхіальну астму, призначили преднізолон (по 5 мг щодня), сальбутамол (інгаляція 2 доз аерозолю 4р. на добу). У зв'язку з проявами судомного синдрому (в анамнезі черепно-мозкова травма) був призначений. б-го розвинулося загострення бронхіальної астми. Із чим це пов'язано?
А. Фенобарбітал прискорив біотрансформацію: а. сальбутамолу, б. преднізолону, Б. Фенобарбітал прискорив екскрецію: а. сальбутамолу, б. преднізолону, В. Фенобарбітал уповільнив екскрецію: а. сальбутамолу, б. преднізолону, Г. сальбутамолу,б.преднізолону
Відповідь: А (б)

74. страждає на ІХС, стенокардією напруги III ФК. ЧСС 90 в хв, АТ 150/80мм. рт. ст. для антиангінальної терапії. а.Нітрати та верапаміл, б. Нітрати та атенолол
в) Нітрати та анаприлін, г) Нітрати та ніфедипін,
д) Ніфедипін та аміодарон
Відповідь: а

75. з приводу стенокардії приймає нітросорбід по 10мг 4р.на добу ЧСС 80 в ми.АД 140/80мм.рт.ст.через 1 міс.після початку терапії знову почастішали напади стенокардії.Які можливі причини погіршення
стану: а)Природний перебіг захворювання; б)Розвиток толерантності до нітратів; в)Синдром міжкоронарного обкрадання; г)Виникнення синдрому рикошету;
Відповідь: а,б

76. відзначаються напади стенокардії при помірних фізичних навантаженнях. В анамнезі колаптоїдний стан після одноразового прийому нітрогліцерину сублінгвально (з тих пір б-ний нітрогліцерин не приймав). Супутні захворювання-ГБ (робочий рівень АТ 160/100мм.рт.
ст., гіпофункція щитовидної залози. На момент огляду АТ 190/100мм.рт.ст., ЧСС 72 за хвилину.
Відповідь: Аміодарон

77. з приводу артеріальної гіпертензії 2 ступеня отримує 0,000075г клофеліну 4р.на добу. У зв'язку з розвитком сенільної депресії призначений меліпрамін. Через 3 дні після призначення меліпраміну у б-ного виник гіпертонічний криз. Напередодні б-ної не приймав клофелін. стану: а) Наслідок природного перебігу захворювання; б) Наслідок гіпертензивного ефекту меліпраміну; в) Наслідок несприятливої ​​взаємодії ЛЗ;
Відповідь: б,в,г

78. з приводу гіпертонічного кризу вводили натрію нітропрусид внутрішньовенно краплинно у великих дозах (зі швидкістю 8 мкг/хв). З'явилися задишка, акроціаноз, болі, що давлять за грудиною, м'язові посмикування. Яка причина погіршення стану б-ного?
Відповідь: Токсична дія ціанідів

79. виявлені часта шлуночкова екстрасистолія та пароксизми миготливої ​​аритмії. ЧСС 74 в хв, АТ 140/80 мм. рт. ст. Останні 3 г. турбують напади стенокардії напруги та спокою.
для подальшого лікування б-ного: а) Хінідин, б) Боннекор, в) Етацизін,
г) Мекситіл, д) Верапаміл, е) Пропранолол
Відповідь: а,б

80. пароксизмальна суправентрикулярна тахікардія на фоні синдрому WPW. Для купірування нападу обрано аймалін. тече-
ня 3-5 хв в 10 мл 5% розчину глюкози або ізотонічного розчину NaCl або в/м, в) 0,5-1 г в/в кожні 2 хв вводять по 0,1-0,2 г або в/м
г) після парентерального введення призначити внутрішньо по 100 мг 4-5 разів на добу, підтримуюча доза 50 мг 3-4 рази на добу.
Відповідь: а,г

81. У б-ної 28л.з діагнозом ВКВ на тлі ХНН з'явилися набряки гомілок, збільшення печінки. При ехокардіографічному дослідженні визначається зниження серцевого викиду. ЧСС 95 хв, АТ 170/100 мм.рт.ст. Які серцеві глікозиди показані хворий?
Відповідь: Дигітоксин

82. У б-ної 28л.з діагнозом ВКВ на тлі ХНН з'явилися набряки гомілок, збільшення печінки. При ехокардіографічному дослідженні визначається зниження серцевого викиду. Хвора приймає дигітоксин. У зв'язку з появою судомного синдрому додатково призначено фенобарбітал (0,3 г на добу). Коли виникнуть зміни у стані б-ної за наявності впливу?
Відповідь: Через 7-14 днів

83. 57 років з приводу постінфарктного атерокардіосклерозу, застійної серцевої недостатності 2 ступеня отримує 40 мг фуросеміду внутрішньовенно та 300 мг
верошпірона внутр. Яку діуретичну терапію Ви призначите б-ному у разі рефрактерності?
Відповідь: Фуросемід 80 мг внутрішньовенно і спіронолактон 300 мг внутрішньо.

84. страждає на неатопічну бронхіальну астму, що супроводжується рясною бронхореєю.Пульс 62 в хв. АТ 140/80 мм.рт.ст.
Відповідь: Атровент

85. наполегливо рецидивуючий синдром бронхіальної обстукції зі зниженою чутливістю до холіно та адренотропних засобів. Бронхіальною астмою страждає більше 10 років.
2-адреностимуляторів більше 6 разів на добу; б) інгаляція м-холіноблокатора;
Відповідь: г, д

86. надійшов з ж-ми на печію, болі в епігастральній ділянці натще, куповані прийомом натрію гідрокарбонату.
функція середньої інтенсивності з низькими лужними резервами,холінергічний тип рецепції. Діагноз: виразкова хвороба 12 п.к. у стадії загострення. Виберіть найбільш ефективне та безпечне ЛЗ та визначте режим його дозування:
Відповідь: Пірензепін до їди по 0,05 г 3 рази на добу протягом 2 днів, потім по 0,05 г 2 рази на добу

87. виявлено дискенезію жовчного міхура за гіпертонічним типом. Виберіть оптимальний варіант лікування.
Відповідь: Но-шпа по 1-2 табл.3 рази на добу, відвар безсмертника по 1/2 склянки за 30 хв до їди

88. страждає хрон. холецистопанкреатитом протягом 5 років.
а) Аллохол, б) Холензим, в) Нікодін, г) Відвар пижми, д) Ксіліт
Відповідь: а,в

89. з суїцидальною метою випила 20 таблеток феназепаму. Через 2 години після прийому препарату доставлена ​​до стаціонару. Виберіть найбільш оптимальні проносні засоби: а) Глауберова сіль, б) Сульфат магнію, в) Екстракт кори жостеру, г) Бісакодил,
д) Касторове масло, е) Морська капуста, ж) Вазелінове масло
Відповідь: а, б, д

90. 46 років надійшов у відділення кардіореанімації з гострим трансмуральним інфарктом міокарда, що виник близько 5 годин тому. до 18-23 хвилин. На 4-й день у б-ного виявлено мікрогема турію (22 еритроцити в полі зору). Яка ваша тактика?
Відповідь: Зменшити дозу гепарину до часу згортання крові не менше 10-12 хв.

91. надійшов з гострим інфарктом міокарда, що виник 5год тому. Призначення: анаприлін 20 мг 4 рази на добу всередину, гепарин внутрішньовенно крапельно по 10 000 ОД кожні 4 години. При цьому вдалося досягти збільшення часу зсідання крові до 18-23 хвилини. На наступний день у б-ного діагностована на правостороння нижньодолева пневмонія. Призначена натрієва сіль бензилпеніциліну (по 1000 000 ОД кожні 4 год) в/в. Через 4 години час згортання крові склало 8 хвилин. Яка ваша тактика?
Відповідь: Змінити шлях введення пеніциліну

92. проведено радикальну операцію з приводу раку шлунка. На 4 добу після операції при дослідженні коагулограми виявлено гіперкоагуляцію та зниження фібринолітичної активності крові. Чи доцільне призначення антикоагулянтів?
Відповідь: Антикоагулянти показані, проте необхідний ретельний контроль для попередження геморагічного синдрому

93. надійшла в стац.з жал-ми на різку слабкість, задишку при ходьбі. При обстеженні в аналізі крові виявлена ​​анемія (гемоглобін-56 г/л), кольоровий показник 1,2, при огляді язика-глосит. У пунктаті кісткового мозку виявлено мегалобластний тип кровотворення. Концентрація заліза в сироватці крові в межах норми -Дефіцитна анамія.Виберіть найбільш оптимальний варіант лікування.
Відповідь: Віт.В12 у дозі 500 мкг/добу через день, фолієва кислота в дозі 1,5 мг/добу, сульфат заліза(80 мгFe2+) 1 раз на добу

94. після переохолодження виникли озноб, підвищення Т.тіла до 38,6С, кашель з відділенням слизово-гнійного мокротиння, болі в правій половині грудної клітини. 2р.на добу вм,гемодез 400 мл внутрішньовенно крапельно, відхаркувальна мікстура по 1ст.л.6р.на добу.Виберіть антиоксидантний препарат,найбільше
що ефективно впливає на процеси вільнорадикального окислення в легенях, який слід додати до терапії, що проводиться
Відповідь: Віт.С

95. з приводу гострого порушення мозкового кровообігу за ішемічним типом давністю 12год отримує реополіглюкін по 400мл внутрішньовенно крапельно
1р.на добу.Виберіть найбільш ефективний у цій ситуації препарат, що володіє антиоксидантними властивостями
Відповідь: Церебролізин

96. протягом 5 років страждає на деформуючий остеоартроз нижніх кінцівок з вираженими синовітами. В анамнезі відзначає лікарську
алергію (на бутадіон, гепарин, метіндол, пеніцилін, теофілін). Через 3 дні у б-ної з'явилися сверблячі еритематозні висипання на шкірі тулуба. Яка найбільш ймовірна причина погіршення стану?
Відповідь: Лікарська алергічна реакція

97. підтверджено діагноз ревматоїдного артриту. Які базисні засоби для лікування ревматоїдного артриту ви можете призначити: а) 4,7-хлорхінолонові препарати (делагіл); ,
д) Препарати золота (кризанол); е) Салазопіридазин,
ж) Антибіотики (тетрацикліни), з) D-пеніциламін,
і) Імуномодулятори (левамізол)
Відповідь: а, б, д, е, з, і

98. Пацієнту з ревматоїдним артритом було призначено метотрексат. Як виявиться дія метотрексату?
Відповідь: Через кілька місяців

99. з ревматоїдним артритом було призначено метотрексат. Які ви вжите заходів щодо контролю за безпекою фармакотерапії метотрексатом у даного пацієнта: а) Щотижневе проведення загального аналізу крові
(краще 2р. на тиждень), б) Проведення аналізу крові з визначенням кількості тромбоцитів кожні 3-4 тижні,
в) Проведення загального аналізу сечі, г) Визначення вмісту сечової кислоти, д) Проведення проби на приховану кров у калі, е) Визначення вмісту трансаміназ, загального білірубіну кожні 6-8 тижнів
Відповідь: а, б, в, д, е

100. з приводу ревматизму тривало отримує делагіл. Які ви вживете заходів щодо контролю безпеки терапії делагілом при його тривалому застосуванні: а) Загальний аналіз крові, б) Загальний аналіз сечі, в) ЕКГ, г) Дослідження очного дна, д) Дослідження полів зору, е )Рентгенологічне дослідження органів грудної клітки, ж)Визначення кількості тромбоцитів, з) Дослідження рогівки
Відповідь: а, б, в, г, д, ж, з

101. 39 років ревматоїдний артрит, переважно суглобова форма, 2 ступені активності. Які варіанти комбінованої терапії доцільно призначити даному пацієнту?
Відповідь: Делагіл по 0,25 г 3 рази на добу, преднізолон по 15 мг на добу, кризан/в по 1 мл 5% розчину 1 раз на тиждень

102. 63 років страждає на цукровий діабет, приймає глібенкламід. Надійшла у відділення з картиною гострої правосторонньої нижньодолової пневмонії, підтвердженої рентгенологічно. Був призначений хлорамфенікол, на який у б-ної відзначалася алергічна реакція. -ний виявлено низький рівень кліренсу креатиніну (24 мл/хв), внаслідок чого цефтріаксон було скасовано. Яким препаратом слід продовжити лікування?
Відповідь: Ципрофлоксацин

103. надійшла у відділення з ж-ми на болі у правій молочній залозі, підвищення Т. до 39,5 С. ​​Захворіла 3 дні тому, на 10-й день після пологів. При вступі у відділення у верхньозовнішньому квадранті правої молоч-
ної залози виявлено гіперемію шкіри, масивний інфільтрат з флуктуацією в центрі. Діагноз: гострий правосторонній мастит. Б-на оперована. Взято посів рани, що відокремлюється. Визначте антибіотик першого вибору
Відповідь: Цефазолін

104. вступила у відділення з картиною гострого правостороннього маститу. Захворіла 3 дні тому, на 10-й день після пологів. Б-на оперована.
Після 2-ї ін'єкції препарату через 20 хв з'явилися зниження артеріального тиску, запаморочення, нудота, блювання, мимовільне сечовипускання, судомний синдром. Яке ускладнення розвинулося у пацієнтки?
Відповідь: Анафілактична реакція

105. Б-на 21г. надійшла у відділення з картиною гострого правостороннього маститу. Захворіла 3 дні тому, на 10-й день після пологів. Б-на оперована. виділення рани виділені
стафілокок, що утворює пеніциліназ, і гемофільна паличка. Виберіть антибактеріальний препарат з урахуванням бактеріальної мікрофлори та особливостей фармакокінетики
Відповідь: Левоміцетин

106. страждає хронічним тонзилітом і хронічним холециститом. Чи потрібно коригувати режим дозування препарату? Якщо так, то яким чином?
Відповідь: Зменшити кратність введення та знизити дозу

107. 50 років звернулася зі скаргами на загальну слабкість, спрагу, часте сечовипускання, свербіж шкіри та зовнішніх статевих органів. При огляді виявлено ожиріння (маса тіла 96 кг при зростанні 168 см). %, реакція ацетон негативна. Які гіпоглікемічні препарати оптимальні у разі?
Відповідь: Бігуаніди

108. 48 років, надійшов зі скаргами на болі, що давлять, що з'являються під час фізичного навантаження, що купуються нітрогліцерином. Три роки тому переніс інфаркт міокарда. У легенях везикулярне дихання. Тони серця приглушені, шум систоли на верхівці, часті екстрасистоли. ЧСС - 92 за хв. АТ - 100/60 мм рт. ст. Печінка не збільшена, набряків немає. ЕКГ – синусова тахікардія, рубцеві зміни міокарда, часта шлуночкова екстрасистолія. Призначено обзидан 160 мг/добу, сустак-форте 19,2 мг/добу, панангін, рибоксин.
Який побічний ефект можливий у хворого при комбінації лікарських засобів?
Відповідь: Гіпотонія, запаморочення.

109. Хворий М., 52 років, надійшов зі скаргами на задишку, серцебиття, біль у правому підребер'ї набряки на ногах. Протягом 18 років перебуває на диспансерному обліку із діагнозом «ревматизм». Шкіра бліда, акроціаноз, рум'янець щік. У базальних відділах легень-незвучні хрипи. Кордони відносної тупості середника розширені вгору та вправо. Тони серця приглушені, аритмічні, на верхівці систолічний шум, акцент? тони на легеневій артерії. Пульс-96 за хв. ЧСС-140 за хв. АТ - 130/85 мм рт. ст. Живіт м'який, печінка на 3-4 см виступає з-під краю реберної дуги. Набряки на ногах. Добовий діурез –650 мл. ЕКГ: відсутній зубець Р, хвилі «F-F», ритм неправильний. Після внутрішньовенного введення 10 мл 10% розчину новокаїнаміду: відновився синусовий ритм з ЧСС - 72 в хв, хворому призначений новокаїнамід внутрішньо по 0,5 г 4 рази на день, дигоксин 0,25 мг 1 табл.
3 рази на добу, фуросемід 40 мг внутрішньо протягом 3-х днів. Через 5 днів у хворого з'явилася нудота, блювання, діарея, запаморочення. ЕКГ: ритм синусовий, ЧСС-76 хв, PQ -0,20 с, QRS-0,1 с. Лікар відмінив дигоксин і фуросемід і призначив унітіол, препарати калію.
Оцініть дії лікаря.
Відповідь: Дії лікаря правильні, тому що не тільки перевищено середню добову дозу дигоксину, але є і взаємодія з новокаїнамідом за зв'язок з білком.

110. У хворого Д., 53 років, діагноз: ІХС, стабільна стенокардія III ФК, постінфарктний кардіосклероз, миготлива аритмія, ХНК? Приймав строфантин, дигоксин, фуросемід, панангін у середніх терапевтичних дозах. Несподівано у хворого підвищилася температура до 38,4 ° С, з'явилася кашель, задишка, крепітація в легенях праворуч. На рентгенограмі легень праворуч у нижній частці визначається ділянка інфільтрації. До лікування додані гентаміцин, сульфокамфокаїн, супрастин.
Виникнення яких побічних ефектів лікування найімовірніше у хворого за такої комплексної терапії?
Відповідь: При комбінації з фуросемідом найімовірніший нефротоксичний ефект гентаміцину.

111. Хворий 28 років надійшов зі скаргами на напади серцебиття, біль голови, озноб. Під час кризу, що розвивається 2-4 рази на рік, підвищується АТ до 260/110 мм рт. ст., ЧСС-140 за хв., з'являються блідість шкіри, пекучі болі в серці, пульсація в голові, іноді підвищення температури тіла до 38. Після нападів поліурія. У період між нападами АТ 120/80 мм рт. ст. При об'єктивному дослідженні органічної патології з боку внутрішніх органів не виявлено. В аналізах крові та сечі без патології.
Вкажіть найбільш ефективний препарат (перший ряд) для усунення кризу у хворого:
Відповідь: Фентоламін.

112. ревматоїдним артритом на тлі курсового лікування метотрексатом виникла виражена носова кровотеча. Чим вона може бути обумовлена: а) Поразкою судин носа внаслідок основного патологічного процесу;
г) Медикаментозно зумовленим зниженням кількості тромбоцитів; д) Токсичним впливом метотрексату на судини носа
Відповідь: г, д

113. У хворого К., 62 років, артеріальна гіпертензія 1 ступеня. Останнє погіршення стану обумовлено психоемоційним стресом. При огляді стан відносно задовільний, незначний головний біль. АТ-170/100 мм рт.ст. («Робочий» АТ-120/70 мм рт.ст), ЧСС-90 за хв. Лікар призначив анаприлін по 60 мгсут, верапаміл 160 мгсут.
Які зміни Ви можете очікувати при призначенні додатково до анаприліну верапамілу?
Відповідь: Посилення негативного дромотропного ефекту.

114. Хворий С., 56 років, щодо стенокардії приймає нітросорбід (10 мг) 1т х 4р на день. Як зміниться тактика проведення антиангінальної терапії, якщо у хворого на фоні лікування нітратами виникне мозковий інсульт?
Відповідь: Скасувати нітрати та призначити антиангінальний препарат з іншої групи.

115. Хворий 42 роки з хронічним гломерулонефритом та артеріальною гіпертензією. При надходженні: АТ 200/120 мм рт ст., Пульс 75-80 уд в хв, набряки на обличчі, попереку, гомілях. Загальний білок сироватки крові 3.8 г%, у сечі білок 16 г/л.
Відповідь: Еналаприл.

116. Який страждає наполегливо рецидивуючим синдромом бронхіальної обструкції лікар ввів п/к 1 мл адреналіну. Які прояви токсичного
дії адреналіну можливі в цій ситуації: а) Порушення ЦНС, б) Екстрасистолія, в) Токсичне ураження печінки, г) Тахікардія, д) Блокада проведення імпульсу по провідній системі серця.
Відповідь: а, б, г

117. 57 років з приводу посинфарктного ардіосклерозу, застійної серцевої недостатності 2Б ступеня отримує 40 мг фуросеміду внутрішньовенно та 300 мг
верошпірона внутр. Яку діуретичну терапію Ви призначите б-ному у разі рефрактерності?
Відповідь: Фуросемід 80 мг внутрішньовенно і спіронолактон 300 мг внутрішньо.

118. страждає на неатопічну бронхіальну астму, що супроводжується рясною бронхореєю.Пульс 62 в хв. АТ 140/80 мм.рт.ст. Після призначення атропіну сульфату у б-ної спочатку відзначалося поліпшення стану-бронхорея різко зменшилася, проте через 10 днів після початку лікування стан знову погіршився: з'явилися лихоманка (37,8 С), задишка, кашель з важко відокремлюваним мокротинням, ЧСС 90 за хвилину. Які причини таких змін стану хворої?
Відповідь: Порушення відходження мокротиння з її подальшим інфікуванням

119. Жінка 52 років страждає на гіпертонічну хворобу?? ст. Приймає резерпін 1 табл. (0.0001) 3 десь у день. АТ нормалізувався через 1 тиждень. Через 4 тижні регулярного прийому з'явилися «голодні» болі в епігастральній ділянці, під час гастроскопії діагностували ерозивний дуоденіт. Чим ви поясните його виникнення?
Відповідь: Підвищення тонусу n vagus на тлі резерпіну та посилення шлункової секреції.

120. Хворому 60 років з ІХС, стабільною стенокардією IV ф.к. призначений кордарон 600 мг/добу (як антиангінальний препарат).
Які побічні ефекти можуть спостерігатись у хворого при тривалому прийомі кордарону?
Відповідь: все перераховане

121. Вибираючи режим дозування лікарських засобів на основі Т?
визначають:
Відповідь: кратність прийому

122. Більш точно характеризує швидкість виведення лікарських засобів із організму:
Відповідь: загальний кліренс

123. Зв'язок лікарських засобів з білками плазми:
Відповідь: визначає можливість розвитку побічних ефектів при поєднанні лікарських засобів

124. Величина біодоступності важлива визначення:
Відповідь: шляхи запровадження лікарських засобів*

125. При тривалому використанні сильних діуретиків може виникнути:
Відповідь: порушення толерантності до глюкози

126. Запаморочення, відсутність відчуття кінцівок, утруднення при посадці та вставанні без візуального контролю та інші симптоми токсичного впливу зустрічаються у 75% пацієнтів, які:
Відповідь: отримують стрептоміцин

127. Передозування симпатоміметиками викликає:
Відповідь: порушення ритму

128. Побічні реакції, пов'язані з антибіотиком моксалактамом, включають наступне:
Відповідь: тромбоцитопенію

129. Комбінований прийом індометацину та гентаміцину найчастіше викликає:
Відповідь: порушення функції нирок

130. Застосування клавуланової кислоти у поєднанні з амоксициліном дозволяє:
Відповідь: розширити спектр дії амоксициліну на штами бактерій, що виробляють бета-лактамазу.

131. Хворому, який довго одержує дифенін, планується додатково призначити інший антиритмічний препарат 1 класу, при призначенні якого антиаритміка потрібно збільшення дозування на 20-30% від стандартного?
Відповідь: всіх препаратів

132. Одночасний прийом внутрішньо тетрацикліну та препаратів Са2+ сприятиме:
Відповідь: зниження всмоктування тетрацикліну

133. Одночасно призначення хлорамфеніколу та аценокумаролу може навести:
Відповідь: до зниження антибактеріальної активності хлорамфеніколу

134. При серцевій недостатності:
Відповідь: дофамін викликає звуження судин кори нирок у високих дозах (понад 10 мкг/кг/хв)

135. Для артеріальної гіпертензії характерно:
Відповідь: збільшення концентрації натрію в судинній стінці

136. Апресин (гідралазин):
Відповідь: викликає тахікардію

137. Бета-адреноблокатори викликають:
Відповідь: зниження ЧСС

138. Справедливі такі твердження про альфа-адреноблокатори:
Відповідь: все вірно

139. Бета-1 - адреноблокатори:
Відповідь: вибірково діють на бета1-адренорецептори, препарати безпечні при бронхіальній астмі

140. Показаннями до застосування бета-адреноблокаторів є:
Відповідь: порушення серцевого ритму

141. Вкажіть правильні твердження:
Відповідь: строфантин значною мірою руйнується в шлунково-кишковому тракті, у зв'язку з чим прийом його внутрішньо нераціональний

142. Показання до призначення СГ:
Відповідь: ХНК у хворих на ІХС, постінфарктний кардіосклероз і постійну форму миготливої ​​тахіаритмії

143. Чинник, що підвищує ризик розвитку інтоксикації СГ:
Відповідь: гіпокаліємія

144. Стан, що підвищує ризик розвитку інтоксикації СГ:
Відповідь: гіпотиреоз

145. Для зменшення ризику розвитку толерантності до нітратів слід:
Відповідь: робити перерви між прийомами препаратів

146. У разі розвитку толерантності до сустака його можна замінити:
Відповідь: корватоном

147. Головний біль може спричинити прийом:
Відповідь: вірні відповіді А,Б,В

148. Подібним за механізмом дії з нітрогліцерином є:
Відповідь: молсидомін

149. При передозуванні яких препаратів може виникнути ортостатична гіпотонія?
Відповідь: нітратів

150. Назвіть групу антиаритміків, які збільшують тривалість потенціалу дії:
Відповідь: блокатори калієвих каналів

151. Який із перерахованих препаратів має найбільш виражену негативну інотропну дію?
Відповідь: дизопірамід

152. Які з некардіальних побічних ефектів притаманні більшості препаратів 1С класу?
Відповідь: порушення зору

153. При терапії дизопірамідом може загостритися таке захворювання:
Відповідь: доброякісна гіперплазія простати з порушенням сечовипускання

154. У яких випадках дозування лідокаїну має бути змінено порівняно зі стандартним?
Відповідь: у пацієнтів із печінковою недостатністю

155. Вкажіть методи контролю за ефективністю застосування діуретиків при набряковому синдромі:
Відповідь: все вірно

156. Вкажіть методи контролю за безпекою застосування діуретиків при набряковому синдромі:
Відповідь: все вірно

157. Вкажіть ефективний та безпечний спосіб поповнення запасів калію в організмі:
Відповідь: призначення панангіна всередину по 2 таблетки 3 рази на день

158. Вкажіть фактори ризику виникнення побічних ефектів «петлевих» діуретиків:
Відповідь: добовий діурез понад 3 л після введення діуретика

159. Вкажіть початок дії спіронолактону:
Відповідь: 4-5 днів

160. Для олужнення сечі використовують такі методи, крім:
Відповідь: цитрат калію 3 мг кожні 6 годин

161. Реакція сечі може бути кислою при використанні таких препаратів, крім:
Відповідь: метіонін

162. Позначте помилкові положення:
Відповідь: немає помилкових положень

163. Розподіліть зазначені препарати за рівнем кумуляції:
Відповідь: неодикумарин

164. Відберіть тверді речовини, які є цілком правильними для препарату стрептокінази:
Відповідь: все вірно

165. Відберіть фактор, що викликає тромбоз або сприяє тромоутворенню:
Відповідь: все вірно

166. Який із наступних лікарських препаратів може зменшити ефект антикоагулянтів непрямої дії?
Відповідь: рифампіцин

167. Які побічні ефекти можуть траплятися при застосуванні гепарину?
Відповідь: все перераховане

168. Хворому з бронхіальною астмою, який тривалий час отримував пролонговані теофіліни, у зв'язку з розвитком інфекції сечовивідних шляхів призначають ципрофлоксацію. В цьому випадку необхідно:
Відповідь: зменшити дозу теофілінів на 30%

169. У дитини, яка довго одержує карбамазепін у зв'язку з наявністю епілепсії, розвивається бронхообструктивний синдром з дихальною недостатністю 2-ї ст. При призначенні амінофіліну такому пацієнту:
Відповідь: доза амінофіліну повинна бути збільшена в 1.5 рази

170. При призначенні теофіліну курцеві:
Відповідь: доза має бути збільшена

171. Вкажіть препарат, який знижує елімінацію теофіліну при одночасному призначенні:
Відповідь: циметидин

172. У хворого на бронхіальну астму, що тривало отримував теотард, на тлі грипозної інфекції та лихоманки з'явилася нудота, блювання, головний біль, безсоння. Менінгіальні симптоми негативні. Терапевтична тактика у разі:
Відповідь: скасувати теотард або зменшити його дозу на 50%

173. Побічні ефекти теофіліну можуть включати такі явища, крім:
Відповідь: розвитку набрякового синдрому

174. Специфічним побічним ефектом, що виникає при застосуванні теофіліну у дітей 1-го року життя, є:
Відповідь: мелена

175. Вкажіть інгаляційний глюкокортикостероїдний препарат, який має найменшу біодоступність:
Відповідь: флутиказону пропіонат

176. Позначте інгаляційний глюкокортикостероїдний препарат, що має найменшу спорідненість з глюкокортикостероїдними рецепторами легень людини:
Відповідь: флутиказону пропіонат

177. Вкажіть препарат, який має найбільший рівень безпеки (за індексом безпеки:)
Відповідь: преднізолон

178. Який із глюкокортикостероїдних препаратів найбільше сприяє розвитку міопатії?
Відповідь: триамцинолон

179. Уповільнення виділення з організму натрію та води, посилення виведення калію (мінералокортикоїдний ефект) більшою мірою характерне:
Відповідь: гідрокортизону

180. Мінералокортикоїдна активність відсутня у:
Відповідь: дексаметазону

181. Виберіть правильну відповідь. Глюкокортикоїди:
Відповідь: є контрінсулярними гормонами

182. При проведенні пульс-терапії кращий:
Відповідь: метилпреднізолон

183. При призначенні на тривалий термін краще використовувати:
Відповідь: преднізолон

184. Який блокатор Н1-гістамінорецепторів протипоказаний при анафілактичному шоці?
Відповідь: дифенгідрамін (димедрол)

185. Виберіть оптимальний блокатор Н1-гістамінорецепторів для лікування алергічного риніту:
Відповідь: азеластин (алергодил)

186. Позначте лікарський засіб із групи стабілізаторів мембран опасистих клітин у лікарській формі у вигляді порошку для інгаляцій:
Відповідь: кромогліцієва кислота (бікромат)

187. До імуностимулятора мікробного походження відноситься:
Відповідь: рибомуніл

188. Основне показання для призначення рибомунілу полягає в:
Відповідь: профілактиці рецидивуючих інфекцій верхніх дихальних шляхів

189. Добре проникають через гематоенцефалічний бар'єр наступні антибактеріальні препарати:
Відповідь: цефалоспорини III генерації

190. Нове покоління макролідних антибіотиків має такі переваги, крім:
Відповідь: нирковий шлях екскреції

191. Фторхінолони відрізняються від хінолонів такими властивостями, крім:
Відповідь: бактеріостатичною дією

192. Зауважте, які твердження щодо цефалоспоринів правильні:
Відповідь: все вірно

193. Наслідки прийому антибіотиків включають:
Відповідь: все вірно

194. Вкажіть препарат вибору при інфекції сечових шляхів, спричиненої синьогнійною паличкою:
Відповідь: цефтазидим

195. Які препарати показані для лікування хламідійної інфекції сечостатевого тракту:
Відповідь: роваміцин

196. Вкажіть препарат із найменш вигідними фармакокінетичними характеристиками:
Відповідь: кетоконазол

197. Вкажіть антимікотичний препарат, який не піддається метаболізму в печінці:
Відповідь: флуконазол

198. Вкажіть антимікотичний препарат (з групи алліламінів), що застосовується в першу чергу для лікування дерматомікозів:
Відповідь: тербінафін

199. Вкажіть клінічний стан, який є показанням для проведення монотерапії НПЗЗ:
Відповідь: позасуглобові ревматичні захворювання (міозит, тендовагініт, синовіт)

200. Для ацетилсаліцилової кислоти характерно:
Відповідь: при оральному прийомі всмоктується переважно з верхнього відділу тонкого кишечника.

201. У порівнянні з індометацином у ацетилсаліцилової кислоти більш виражено:
Відповідь: антиагрегантна дія на тромбоцити

202. На швидкість виведення ацетилсаліцилової кислоти та її метаболітів впливають:
Відповідь: рівень рН сечі

203. Шлунково-кишкові ускладнення при застосуванні ацетилсаліцилової кислоти пов'язані з:
Відповідь: все перераховане

204. Для фенілбутазону характерно:
Відповідь: все вірно

205. При взаємодії індометацину з іншими препаратами:
Відповідь: зменшується діуретична активність фуросеміду

206. Які побічні реакції НПЗЗ коригує комплексний препарат артротек (диклофенак натрію + мізопростол)
Відповідь: НПЗЗ-гастропатії

207. Які характеристики парацетамолу висунули цей препарат на перше місце у ряді анальгетиків-антипіретиків?
Відповідь: більш ранній настання аналгетичного та жарознижувального ефекту

208. Виберіть препарат, що селективно інгібує циклооксигеназу2:
Відповідь: мелоксикам

209. Найкращий аналгезуючий ефект фентанілу спостерігається в комбінації з:
Відповідь: дроперидолом

210. Назвіть протизапальний засіб пролонгованої дії:
Відповідь: піроксикам

211. У Б-ої 52л.розвинулася картина гіпертонічного кризу 2 типу з ЧСС 62 в хв.АД 200/140 мм.рт.ст.У легенях велика кількість вологих хрипів.
Відповідь: Фуросемід

212. протягом 6 років страждає на цукровий діабет 1 типу, отримує інсулін по 54 ОД/добу, що підтримує рівень глікемії в межах 7,0 ммоль/л. Останнім часом у зв'язку з підвищенням АТ до 16090 мм.рт.ст. Лікар призначив гіпотіазид у добовій дозі 75 мг у комбінації з еналаприлом у дозі 5 мг. Через 10 днів у хворого рівень цукру в крові 10,5 ммольл, відзначається погіршення самопочуття. Що є провідною причиною зміни рівня цукру у крові?
Відповідь: Комбінація еналаприлу з гіпотіазидом

213. розвинулася судомна форма гіпертонічного кризу, стан тяжкий, цифри АТ 200120 мм.рт.ст., ЧСС 120 за хв. З призначення якого препарату слід розпочинати терапію?
Відповідь: Діазепама

214. на фоні застосування антибіотика цефтріаксону протягом 10 днів розвинулася картина псевдомембранозного коліту. Який перший крок алгоритму лікарської допомоги?
Відповідь: скасування цефтріаксону, призначення ванкоміцину або метронідазолу

215. з приводу загострення виразкової хвороби шлунка у комплексі терапії призначено кларитроміцин. Назвіть основні ознаки препарату від еритроміцину.
Відповідь: все вірно

216. після перенесеної операції на черевній порожнині на 4 добу розвинулася лівостороння нижньодолева пневмонія. Результати експрес-аналізу показали наявність MRSA, пеніциліно- та аміноглікозидорезистентних штамів ентерококів. Препарати вибору:
Відповідь: Ванкоміцин

217. знаходиться у відділенні інтенсивної терапії з приводу синьогнійної інфекції. Виберіть препарати 1 ряду для лікування?
Відповідь: Цефтазидим + аміноглікозиди

218. 40 років без супутніх захворювань із приводу позалікарняної пневмонії в амбулаторних умовах було призначено спіраміцин внутрішньо по 3 млн. МО 2 рс, на 2 добу лікування відзначалися інтенсивні гастралгії, нудота, одноразове блювання. Виберіть альтернативний препарат.
Відповідь: доксициклін

219. з хронічним обструктивним бронхітом виявлено пневмонію середньої тяжкості, в амбулаторних умовах призначено амоксиклав внутрішньо по 625 мг 3 рс. На 2 добу у хворого розвинулася кропив'янка, бронхоспазм. Назвіть альтернативний препарат для лікування пневмонії.
Відповідь: моксифлоксацин усередину

220. У ВІЛ-інфікованого б.44 років діагностовано пневмоцистну пневмонію. Назвіть препарат для лікування?
Відповідь: ко-тримоксазол в 20 мгкгс 4 рс протягом 21 добу

221. У б-ної 28л. відзначаються щоденні симптоми бронхіальної астми, часті загострення, часті нічні симптоми, діагностовано тяжку персистуючу бронхіальну астму. Назвіть препарати базової терапії.
Відповідь: інгаляційні глюкокортикоїди (більше 1000 мкг беклометазону дипропіонату) + інгаляційні бета-2-агоністи тривалої дії

222. До лікаря звернулася вагітна жінка (термін вагітності 6-7 тижнів) з клінічними ознаками гострої пневмонії. Які групи антибактеріальних препаратів дозволені для застосування у вагітних?
Відповідь: цефалоспорини

223. 57 років з приводу артеріальної гіпертензії помірного ступеня отримує у монорежимі інгібітор АПФ – еналаприл. У хворого через 2 роки прийому препарату спостерігається недостатність ефекту. Який найприйнятніший варіант оптимізації терапії?
Відповідь: додавання до препарату діуретика (гіпотіазиду або індапаміду)

224. отримує антибактеріальний препарат щодо інфекційного процесу. При внутрішньовенній інфузії препарату відзначається реакція у вигляді вираженого почервоніння шкірних покривів верхньої половини тулуба, обличчя, шиї, симптоми, що значно зменшуються при зниженні швидкості інфузії. На який препарат наголошується така реакція?
Відповідь: Ванкоміцин

225. У вагітної жінки відзначається активація ревматичного процесу. Який препарат із групи антикоагулянтів можна призначити вагітною?
Відповідь: Гепарин

226. надійшов з ж-ми підвищення АТ до цифр 15090 мм.рт.ст. на тлі психоемоційного перенапруги, серцебиття, тривогу, порушення сну. Рік тому виявлено цукровий діабет 2 типу, що отримує манініл. Назвіть препарат для лікування АГ.
Відповідь: Атенолол

227. у зв'язку з симптомами стенокардії та порушення ритму призначені препарати: анаприлін 200 мгс та верапаміл 240 мгс протягом тривалого часу. Які можливі побічні реакції?
Відповідь: Розвиток а-в блокад, брадикардія

228. Жінка 34 років приймає естрогеновмісні контрацептивні препарати. Лікар призначив доксициклін у дозі 200 мг протягом 2 тижнів. Яка ймовірна взаємодія очікується?
Відповідь: Знижується ефект контрацепції

229. Анестезіолог при введенні кетаміну з профілактичною метою призначив хворому на діазепам. Профілактика якого стану проводиться в такий спосіб?
Відповідь: Постнаркозних галюцинацій

230. 46 років вступив із картиною гострого деструктивного апендициту. Препарат вибору антибіотикопрофілактики?
Відповідь: Цефазолін

231. До лікаря звернувся хворий зі скаргами на кашель, підвищення температури до 39 С, біль у грудній клітці. Діагностовано правобудову бронхопневмонію. Призначений препарат протягом 3 діб, що має постантибіотичний ефект. Назвіть препарат вибору.
Відповідь: Азітроміцин

232. з приводу гострого правостороннього пієлонефриту призначено цефазолін по 2 гс протягом 10 днів. Назвіть найбільш типову помилку під час вибору даного антибіотика
Відповідь: Недостатньо висока активність щодо грамнегативної флори

233. одночасно отримує протягом 14 днів фторхінолоновий антибіотик - офлоксацин з приводу інфекції сечовивідних шляхів та диклофенак натрію у зв'язку з суглобовим синдромом. Яка ймовірна взаємодія очікується?
Відповідь: підвищення ризику порушення ЦНС та розвитку судом

234. після переохолодження виникли озноб, підвищення Т. тіла до 38,6С, кашель з відділенням слизово-гнійного мокротиння, болі в лівій половині грудної клітини. Клінічно і рентгенологічно встановлено діагноз лівосторонньої нижньодолової пневмонії. на добу вм, гемодез 400 мл внутрішньовенно крапельно, відхаркувальна мікстура по 1ст.л.6р.на добу. На 3 добу відзначалася реакція у вигляді кропив'янки, свербежу. Виберіть антибактеріальний препарат для заміни?
Відповідь: Спіраміцин

235. Під час проведення наркозу анестезіолог з метою антибіотикопрофілактики призначив антибактеріальний препарат. У хворого розвинулася зупинка дихання. Який препарат використовувався?
Відповідь: Гентаміцин

236. протягом 10 років страждає на деформуючий остеоартроз нижніх кінцівок з вираженими синовітами. В анамнезі відзначає лікарську алергію на бутадіон. У відділенні б-ної призначені реопірин по 5 мл в/м 1 раз на добу. Через день у б-ної з'явилися сверблячі еритематозні висипання на шкірі тулуба.
Відповідь: Лікарська алергічна реакція

237. з приводу порушення ритму серця призначено новокаїнамід, одночасно у зв'язку з сезонним алергічним ринітом призначено цетиризин. Яка ймовірна взаємодія очікується?
Відповідь: важкі форми аритмії (типу піруету)

238. Пацієнту із ВКВ було призначено метотрексат. Через який час очікується стабільний прояв лікувального ефекту?
Відповідь: бірнеше айдан кейін

239. з гіпертонічним кризом призначено препарат фозиноприл. Незважаючи на прийом адекватної дози препарату не відмічено зниження рівня артеріального тиску в найближчі хвилини та години. Назвіть причину.
Відповідь: При кризах не використовуються депопрепарати

240. з метою планової терапії АГ був призначений апресин протягом тривалого часу. У хворого через місяць застосування препарату стали відзначатись серцебиття, стенокардитичні болі, зниження ефекту від лікування. Назвіть основну причину явищ, що розвинулися
Відповідь: Апресин не призначається для планової терапії АГ

241. 42 років вступив у відділення інтенсивної терапії з приводу тяжкого гіпертонічного кризу. Протягом 5 діб внутрішньовенно інфузійно вводився нітропрусид натрію. На 6-ту добу у хворого розвинулася картина інтоксикації у вигляді невгамовного блювання, зниження функції серцево-судинної, дихальної та видільної систем. Назвіть основну причину стану, що розвинувся.
Відповідь: Передозування препарату (накопичення тіоціанатів у крові)

242. 54 років страждає на цукровий діабет 1 типу, приймає пролонгований інсулін. Надійшла у відділення з картиною гострої правобічної нижньодольової пневмонії, підтвердженої рентгенологічно. Був призначений цефтріаксон, на який у б-ної відзначалася алергічна реакція. амікацином.Однак при обстеженні у б-ної виявлено низький рівень кліренсу креатиніну (30 мл/хв), внаслідок чого амікацин був скасований. Яким препаратом слід продовжити лікування?
Відповідь: Спіраміцин

243. надійшла у відділення з ж-ми на болі в низу живота, підвищення Т. до 39,5 С. ​​Захворіла 2 дні тому, на 6-й день після пологів. При гінекологічному огляді – картина гострого післяпологового ендометриту. Результати бакпосіву: золотистий стафілокок, що утворює пеніциліназ, протей. Визначте антибіотик першого вибору
Відповідь: Цефепім

244. 25 л. надійшла у відділення з картиною гострого правостороннього пієлонефриту. Захворіла 3 дні тому після переохолодження. Після 2-ї ін'єкції препарату через 10 хв з'явилися зниження артеріального тиску, запаморочення, нудота, блювання, мимовільне сечовипускання, судомний синдром. Яке ускладнення розвинулося у пацієнтки?
Відповідь: Анафілактична реакція

245. Б-на 28 л. надійшла у відділення з картиною гострого правостороннього маститу. Захворіла на 12-й день після пологів. Б-на оперована. Виберіть антибактеріальний препарат з урахуванням бактеріальної мікрофлори та особливостей фармакокінетики.
Відповідь: Оксацилін + флуконазол

246. страждає хронічним холециститом. У посіві жовчі при обстеженні виявлені золотистий стафілокок та кишкова паличка. В анамнезі відмічено алергію на оксацилін. Препарати вибору.
Відповідь: Цефтріаксон

247. 58 років звернулася зі скаргами на загальну слабкість, спрагу, часте сечовипускання, свербіж шкіри та зовнішніх статевих органів. При огляді: маса тіла 56 кг при зростанні 168 см). %, реакція ацетон негативна. Які гіпоглікемічні препарати оптимальні у разі?
Відповідь: Препарати сульфонілсечовини

248. 53 років, надійшов зі скаргами на серцебиття, перебої, часом задишку. Ці явища стали непокоїти після перенесеного інфаркту міокарда 2 роки тому. Прийом новокаїнаміду протягом 3 місяців приніс значне полегшення. Проте останнім часом самопочуття погіршилося. Подальша тактика ведення хворого.
Відповідь: повне обстеження та вибір препарату

249. 33 років щодо післяопераційного гнійного перитоніту отримує комбінацію препаратів: цефтріаксон + амікацин + метронідазол. В анамнезі – жовчнокам'яна хвороба. Назвіть препарат, який повинен призначатися хворому.
Відповідь: Цефтріаксон

250. 45 років з приводу кандидозної пневмонії був призначений флуконазол протягом 3 днів, потім усередину. На 4-ту добу лікування відзначено реакцію у вигляді сильного головного болю, нудоти. Вирішено було замінити препарат на кетоконазол. Оцініть адекватність тактики.
Відповідь: Кетоконазол не є адекватною заміною через невигідні фармакокінетичні характеристики.

251. 42 років, надійшов зі скаргами на виражене серцебиття, підвищення цифр АТ до 240-140 мм.рт.ст. Під час обстеження виявлено значне підвищення у крові рівня катехоламінів. Назвіть препарат вибору для усунення кризу.
Відповідь: фентоламін

252. 50 років, вступив із картиною гострого лівостороннього пієлонефриту. Були призначені цефазолін + гентаміцин у середньотерапевтичних дозах. У хворого при обстеженні кліренс креатиніну – 50 млмін. Які можливі наслідки терапії?
Відповідь: Ризик нефротоксичності

253. 48 років з пароксизмальною над- та шлуночковою тахікардією був призначений кордарон ст в першу добу, потім усередину. При обстеженні виявлено порушення функції щитовидної залози, а блокада 2-3 ступеня. Подальша тактика.
Відповідь: Відміна препарату, призначення новокаїнаміду

254. 50 років перебуває у відділенні кардіореанімації щодо гострого інфаркту міокарда, отримує комплекс терапії. Які основні параметри моніторингу для призначення прямих антикоагулянтів?
Відповідь: АЧТВ, час зсідання крові, сеча на еритроцити

255. У відділенні перебуває хворий після операції з приводу апендициту. Лікар призначив гентаміцин по 80 мг 3 рс вм. Пацієнт 2 роки тому переніс гострий гломерулонефрит, нині кліренс креатиніну становить 50 млмін. Яка корекція потрібна?
Відповідь: заміна на препарат, що не має нефротоксичності

256. 56 років отримує дигоксин по 0,25 гс протягом останнього року. В даний час стало відзначатись підвищення цифр АТ до 180110 мм.рт.ст. Лікарем призначений лізиноприл у дозі 10 мгс. Через 3 місяці у пацієнта діагностовано дигіталісну інтоксикацію. Подальша тактика.
Відповідь: призначення гіпотензивного препарату іншого ряду

257. 53 років діагностовано кандидозний та аспергільозний менінгіт. Препарати вибору.
Відповідь: Амфотерицин В

258. 58 років, що отримує тривалий час глюкокортикоїди до комплексу терапії був включений синтетичний антибіотик широкого спектра у зв'язку з інфекцією репродуктивної сфери. На 14 добу спільного застосування відмічено тяжке ускладнення у вигляді розриву ахілового сухожилля. Назвіть антибіотик, який у поєднанні з кортикостероїдами викликав дане ускладнення.
Відповідь: Левофлоксацин

259. у відділення надійшов Б.43 років із картиною гострої хламідійної пневмонії. Назвіть препарати вибору.
Відповідь: Роваміцин

260. У хворого 24 років діагностовано неускладнену форму гонореї. Назвіть препарат вибору.
Відповідь: Цефтріаксон

261. Виберіть правильне твердження: а) біодоступність-кількість ЛЗ, що надходить у системний кровотік, виражене у відсотках від введеної дози, б) біодоступність визначається величиною адсорбції ЛЗ у ШКТ і виро-
біодоступність визначають за формулою: F = AUC (в/м або всередину)/AUC (в/в).г)біодоступність ЛЗ при внутрішньом'язовому введенні визначається ступенем його всмоктування та біотрансформації в організмі.
Відповідь: а Б В

262. страждає на неатопічну бронхіальну астму, що супроводжується рясною бронхореєю.Пульс 62 в хв. АТ 140/80 мм.рт.ст.
Відповідь: Атровент

263. Механізми всмоктування ЛЗ у кишечнику:
а) пасивна дифузія; б) фільтрація; в) активний транспорт; г) полегшений транспорт; д) піноцитоз:
Відповідь: а, д

264. надійшов з ж-ми на печію, болі в епігастральній ділянці натще, куповані прийомом натрію гідрокарбонату.
функція середньої інтенсивності з низькими лужними резервами,холінергічний тип рецепції. Діагноз: виразкова хвороба 12 п.к. у стадії загострення. Виберіть найбільш ефективне та безпечне ЛЗ та визначте режим його дозування:
Відповідь: Пірензепін до їди по 0,05 г 3 рази на добу протягом 2 днів, потім по 0,05 г 2 рази на добу

265. виявлено дискенезію жовчного міхура за гіпертонічним типом. Виберіть оптимальний варіант лікування.
Відповідь: Но-шпа по 1-2 табл.3 рази на добу, відвар безсмертника по 1/2 склянки за 30 хв до їди

266. страждає хрон. холецистопанкреатитом протягом 5 років.
а) Аллохол, б) Холензим, в) Нікодін, г) Відвар пижми,
д) Ксіліт
Відповідь: а,в

267. з суїцидальною метою випила 20 таблеток феназепаму. Через 2 години після прийому препарату доставлена ​​до стаціонару. Виберіть найбільш оптимальні проносні засоби: а) Глауберова сіль, б) Сульфат магнію, в) Екстракт кори жостеру, г) Бісакодил,
д) Касторове масло, е) Морська капуста, ж) Вазелінове масло
Відповідь: а, б, д

268. 46 років надійшов у відділення кардіореанімації з гострим трансмуральним інфарктом міокарда, що виник близько 5 годин тому. до 18-23 хвилин. На 4-й день у б-ного виявлено мікрогематурію (22 еритроцити в полі зору). Яка ваша тактика?
Відповідь: Зменшити дозу гепарину до часу згортання крові не менше 10-12 хв.